Nclex-Pn Respiratory

Lakukan tugas rumah & ujian kamu dengan baik sekarang menggunakan Quizwiz!

The chest x-ray report for a client states that the client has a left apical pneumothorax. The nurse should monitor the status of breath sounds in that area by placing the stethoscope in which location? 1.Near the lateral twelfth rib 2.Just under the left clavicle 3.In the fifth intercostal space 4.Posteriorly under the left scapula

2 (Rationale: The apex of the lung is the rounded, uppermost part of the lung. To check breath sounds in a client with a left apical pneumothorax, the nurse would place the stethoscope just under the left clavicle. The other options are incorrect.)

The nurse is caring for a hospitalized client with a suspected diagnosis of tuberculosis (TB). Which finding does the nurse expect to note during data collection? 1.High fever 2.Chills and night sweats 3.Complaints of diarrhea 4.Petechiae on the upper extremities

2 (Rationale: The client with tuberculosis usually experiences cough (either productive or nonproductive), fatigue, anorexia, weight loss, dyspnea, hemoptysis, chest discomfort or pain, chills and sweats (which may occur at night), and a low-grade fever.)

The nurse is caring for a client after a bronchoscopy and biopsy. Which finding should be reported *immediately* to the primary health care provider (PHCP)? 1.Dry cough 2.Hematuria 3.Bronchospasm 4.Blood-tinged sputum

3 (Rationale: If a biopsy was performed during a bronchoscopy, blood-streaked sputum is expected for several hours. Frank blood indicates hemorrhage. A dry cough may be expected. The client should be assessed for signs of complications, which would include cyanosis, dyspnea, stridor, bronchospasm, hemoptysis, hypotension, tachycardia, and dysrhythmias. Hematuria is unrelated to this procedure.)

A client who underwent a bronchoscopy was returned to the nursing unit 1 hour ago. The nurse determines that the client is experiencing complications of the procedure if the nurse notes which sign/symptom? 1.Oxygen saturation of 95% 2.Weak gag and cough reflex 3.Respiratory rate of 22 breaths per minute 4.Breath sounds greater on the right than the left

4 (Rationale: Asymmetrical breath sounds could indicate pneumothorax, and this should be reported to the primary health care provider. A weak cough and gag reflex 1 hour postprocedure is an expected finding because of residual effects of intravenous sedation and local anesthesia. A respiratory rate of 22 breaths per minute and an oxygen saturation of 95% are acceptable measurements.)

The nurse is monitoring a client for Biot's respirations. Which condition causes Biot's respirations? 1.Emphysema 2.Renal failure 3.Severe anxiety 4.Neurological disorders

4 (Rationale: Biot's respirations are irregular respirations of varying depth and periods of apnea seen in neurological disorders. Emphysema causes obstructive breathing, rising end-expiratory level with forced rapid breathing. Renal failure can cause Kussmaul respirations, which are rapid and deep. Severe anxiety causes hyperventilation.)

The nurse is caring for the client who is at risk for lung cancer because of an extremely long history of heavy cigarette smoking. The nurse tells the client to report which frequent *early* symptom of lung cancer? 1.Hoarseness 2.Pleuritic pain 3.Hemoptysis 4.Nonproductive hacking cough

4 (Rationale: Cough is the most frequent early symptom of lung cancer; it begins as nonproductive and hacking and progresses to productive. In the smoker who already has a cough, a change in the character and frequency of the cough usually occurs. Hoarseness and blood-streaked sputum are later signs. Pain is a very late sign and is usually pleuritic in nature.)

A client is admitted to the hospital with acute exacerbation of chronic obstructive pulmonary disease (COPD) and has an arterial blood gas test done. Which results would the nurse expect to note? 1.Po2 of 70 mm Hg and Pco2 of 45 mm Hg 2.Po2 of 68 mm Hg and Pco2 of 40 mm Hg 3.Po2 of 62 mm Hg and Pco2 of 40 mm Hg 4.Po2 of 60 mm Hg and Pco2 of 50 mm Hg

4 (Rationale: During an acute exacerbation of COPD, the arterial blood gases deteriorate with a decreasing Po2 and an increasing Pco2. In the early stages of COPD, arterial blood gases demonstrate a mild to moderate hypoxemia, with the Po2 in the high 60s to high 70s and normal arterial Pco2. As the condition advances, hypoxemia increases and hypercapnia may result.)

The nurse checks the water seal chamber of a closed chest drainage system and notes fluctuations in the chamber. Based on this finding, the nurse makes which determination? 1.An air leak is present. 2.The tubing is kinked. 3.The lung has reexpanded. 4.The system is functioning as expected.

4 (Rationale: Fluctuations in the water seal chamber are normal during inhalation and exhalation until the lung reexpands and the client no longer requires chest drainage. If fluctuations are absent, it could indicate an air leak, kinking, or that the lung has reexpanded.)

The nurse is preparing a plan of care for the client who will be returning from surgery following a right lung wedge resection. Included in the plan of care is that in the postoperative period, the nurse should avoid which positioning? 1.In low-Fowler's 2.In semi-Fowler's 3.On the left side 4.On the right side

4 (Rationale: Following a wedge resection, the client should not be placed on the operative side. Lying on the operative side hinders expansion of remaining lung tissue and may accentuate perfusion of poorly ventilated tissue. This further impedes normal gas exchange. In addition, complete lateral turning may be contraindicated. The surgeon's prescriptions for positioning after this surgical procedure are always followed.)

The nurse is teaching a client with chronic airflow limitation (CAL) about positions that help breathing during dyspneic episodes. The nurse instructs the client to avoid which position because it will aggravate breathing? 1.Sitting up and leaning on a table 2.Standing and leaning against a wall 3.Sitting up with elbows resting on knees 4.Lying on his or her back in low-Fowler's position

4 (Rationale: The client should use the positions outlined in options 1, 2, and 3. These positions allow for maximal chest expansion and decreased use of accessory muscles of respiration. The client should not lie on his or her back because it reduces movement of a large area of the client's chest wall. Sitting is better than standing whenever possible. If no chair is available, then leaning against a wall while standing allows accessory muscles to be used for breathing rather than posture control.)

The nurse is told that an assigned client will have the chest tubes removed. The nurse plans to do which in preparation for the procedure? 1.Clamp the chest tubes. 2.Empty the drainage system. 3.Disconnect the drainage system. 4.Administer pain medication 15 to 30 minutes before the procedure.

4 (Removal of chest tubes can be uncomfortable for a client. The nurse should medicate the client 15 to 30 minutes before the chest tube is removed. The remaining options are inappropriate actions and would not be performed by the nurse.)

A client at risk for pulmonary embolism (PE) suddenly develops respiratory distress, chest pain, and anxiety. The nurse should plan to take which actions? *Select all that apply.* 1.Check vital signs. 2.Administer warfarin. 3.Notify the registered nurse. 4.Begin low-flow oxygen therapy. 5.Raise the bed to a low-Fowler's position.

1, 3 ,4

The nurse is assisting in planning care for a client scheduled for insertion of a tracheostomy. Which equipment should the nurse plan to have at the bedside when the client returns from surgery? 1.Obturator 2.Oral airway 3.Epinephrine 4.Tracheostomy tube with the next larger size

1 (Rationale: A replacement tracheostomy tube of the same size and an obturator is kept at the bedside at all times in case the tracheostomy tube is dislodged. In addition, a curved hemostat that could be used to hold the trachea open, if dislodgment occurs, should also be kept at the bedside. An oral airway and epinephrine would not be needed.)

The nurse is performing tracheal suctioning on an assigned client. The nurse uses which parameter as the accurate indicator that suctioning has been *effective*? 1.Breath sounds are now clear. 2.Suctioning is required only once a shift. 3.Oxygen saturation has increased two points. 4.Respiratory rate has gone down by four breaths per minute.

1 (Rationale: Clear breath sounds are the most accurate indicator of the effectiveness of a suctioning procedure. Options 3 and 4 are incorrect because they are less precise. Option 2 is incorrect because the need for suctioning may be influenced by factors other than the effectiveness of previous suctioning. These other factors could include improvement of underlying respiratory condition, fluid status, and effectiveness of cough.)

The nurse is assigned to assist in caring for a client diagnosed with a pneumothorax who has a chest tube connected to a closed-chest drainage system. The client asks the nurse why a chest tube was inserted. Which response by the nurse explains the purpose of a chest tube? 1."To help lessen any discomfort." 2."To allow for reexpansion of the lung." 3."It will help prevent any lung infections." 4."To prevent further damage to the lung."

2 (Rationale: A chest tube may be inserted after a pneumothorax and connected to water-seal drainage to remove the air and allow reexpansion of the lung. It does not lessen discomfort, prevent further damage to the lung, or help prevent lung infections.)

The nurse is caring for a client who is suspected of having lung cancer. The nurse monitors the client for which *most* frequent *early* 1.Cough 2.Wheezing 3.Pleuritic pain 4.Blood-streaked sputum sign of lung cancer?

1 (Rationale: Cough is the most frequent early sign of lung cancer that begins as nonproductive and hacking and progresses to productive. In the smoker who already has a cough, a change in the character and frequency of the cough usually occurs. Wheezing and blood-streaked sputum are later signs. Pain is a very late sign and is usually pleuritic in nature.)

A client begins to drain small amounts of red blood from a tracheostomy tube 36 hours after a supraglottic laryngectomy. The licensed practical nurse should perform which action? 1.Notify the registered nurse. 2.Increase the frequency of suctioning. 3.Add moisture to the oxygen delivery system. 4.Document the character and amount of drainage.

1 (Rationale: Immediately following laryngectomy, there is a small amount of bleeding from the tracheostomy that resolves within the first few hours. Bleeding 24 hours after the surgery may be a sign of impending rupture of a vessel. The bleeding in this instance represents a potentially life-threatening situation, and the registered nurse needs to be notified who will then contact the primary health care provider. Although the other options may be appropriate, they do not address the urgency of the problem. Failure to notify the primary health care provider in a timely fashion places the client at risk.)

The nurse is caring for a client at home who has had a tracheostomy tube for several months. The nurse monitors the client for complications associated with the long-term tracheostomy and suspects tracheoesophageal fistula if which observation is noted for the client? 1.Abdominal distention 2.Purulent drainage around the tracheotomy site 3.Excessive secretions from the tracheotomy site 4.Inability to pass a suction catheter through the tracheotomy

1 (Rationale: Necrosis of the tracheal wall can lead to an artificial opening between the posterior trachea and the esophagus. This problem is called tracheoesophageal fistula. The fistula allows air to escape into the stomach, causing abdominal distention. It also can cause aspiration of gastric contents. Option 2 may indicate an infection. Option 3 may indicate the need for more frequent suctioning. Option 4 may indicate an obstruction of some sort or the presence of bronchoconstriction.)

A client experiencing a pleural effusion had a thoracentesis. Analysis of the extracted fluid revealed a high red blood cell count. The nurse interprets that this result is consistent with which diagnosis? 1.Trauma 2.Infection 3.Liver failure 4.Heart failure (HF)

1 (Rationale: Pleural effusion that has a high red blood cell count may result from trauma and may be treated with placement of a chest tube for drainage. Other causes of pleural effusion include infection, HF, liver or renal failure, malignancy, or inflammatory processes.)

A client with a suspected throat infection with Streptococcus needs to have a throat culture obtained. The nurse should take which action after obtaining the culture if the specimen cannot be delivered to the laboratory for at least an hour? 1.Refrigerate the specimen. 2.Obtain a second specimen immediately. 3.Discard the specimen and make the client wait an hour to get a new one. 4.Keep the client nothing-by-mouth (NPO) for 30 minutes and obtain a second specimen.

1 (Rationale: Refrigeration will stabilize the culture and prevent the growth of additional bacteria. Options 2, 3, and 4 are unnecessary.)

The nurse is working in a tuberculosis (TB) screening clinic. The nurse understands that which population is at highest risk for TB? 1.Residents of a long-term care facility 2.Persons admitted to the hospital for day surgery 3.A family who has recently emigrated from Australia 4.Children older than 6 years of age in a summer school program

1 (Rationale: Residents of long-term care facilities are considered high-risk candidates for TB. Children younger than 4 years of age also are considered a high-risk group. Persons admitted for day surgery are not high-risk candidates. Foreign immigrants (especially from Mexico, the Philippines, and Vietnam) are considered high risk, but those from Australia are not.)

The nurse is discussing signs of severe airway obstruction with a group of nursing students. Which sign should the nurse emphasize as one that indicates severe airway obstruction? 1.Cyanosis 2.A loud cough 3.Pink color to the skin 4.Respiratory rate of 12 to 16 breaths per minute

1 (Rationale: Signs of severe airway obstruction include cyanosis, poor air exchange, increased breathing difficulty, a silent cough, or inability to speak or breathe. Options 2, 3, and 4 are incorrect and may be signs of mild respiratory distress that would not require immediate intervention.

The nurse is caring for the client diagnosed with tuberculosis (TB). Which finding made by the nurse would be inconsistent with the usual clinical presentation of tuberculosis? 1.High-grade fever 2.Chills and night sweats 3.Anorexia and weight loss 4.Nonproductive or productive cough

1 (Rationale: The client with TB usually experiences cough (either productive or nonproductive), fatigue, anorexia, weight loss, dyspnea, hemoptysis, chest discomfort or pain, chills and sweating (which may occur at night), and a low-grade fever.)

The nurse is collecting data on a client admitted to the hospital with suspected carbon monoxide poisoning and notes that the client behaves as if intoxicated. The nurse uses this data to make which interpretation? 1.The behavior is likely the result of hypoxia. 2.The client probably suffers from alcoholism. 3.The client must also have a high blood alcohol level. 4.The carbon monoxide has caused the blood glucose to fall.

1 (Rationale: The client with carbon monoxide poisoning may appear intoxicated. This is the end result of hypoxia on the central nervous system (CNS). With carbon monoxide poisoning, oxygen cannot easily bind onto the hemoglobin, which is carrying strongly bound carbon monoxide. Because cerebral tissue has a critical need for oxygen, sustained hypoxia may yield this typical finding. For this reason, options 2, 3, and 4 are incorrect interpretations.)

The nurse is assisting in caring for the client immediately after removal of the endotracheal tube following radical neck dissection. The nurse interprets that which sign experienced by the client should be reported *immediately* to the registered nurse (RN)? 1.Stridor 2.Lung congestion 3.Occasional pink-tinged sputum 4.Respiratory rate of 26 breaths per minute

1 (Rationale: The nurse reports the presence of stridor to the RN immediately. This is a high-pitched, coarse sound that is heard with the stethoscope over the trachea. It indicates airway edema and places the client at risk for airway obstruction. A respiratory rate of 26 breaths per minute and congestion are abnormal, but additional data are needed to determine if these pose a serious problem at this time. Occasional pink-tinged sputum may be expected at this time.)

A client with chronic obstructive pulmonary disease (COPD) on bed rest is weaned from the ventilator before transferring to a medical unit. To adequately restore client strength before getting the client out of bed, which is the *priority* client activity for the nurse to incorporate in the plan of care? 1.Instruct the client to reposition himself. 2.Elevate the head of the bed to 15 degrees. 3.Transfer the client to the chair three times daily. 4.Perform passive flexion and extension of the ankles.

1 (Rationale: Therapy for COPD usually includes glucocorticoids that carry a high risk of complications such as muscle and bone wasting, fragile skin, impaired immune functioning, and fluid retention, so the nurse must restore some client strength before attempting to get the client out of the bed. Because the client is likely to be weak from bed rest and lack of activity during mechanical ventilation and treatment, the nurse establishes outcomes for the client including restoration of pulmonary, cardiovascular, and musculoskeletal functioning to return to baseline functioning. To begin safely, the nurse instructs the client to reposition himself in bed to exert force on muscles and bones helping to reverse the tissue loss incurred during bed rest. The nurse initially positions a client with COPD at 45 degrees or higher until the client can tolerate a lower position and still maintain adequate oxygenation. Self-repositioning can be followed with dangling at the bedside and sitting in the chair before attempting ambulation. Active flexing and extending ankles is very important to prevent a thromboembolic event related to bed rest, but this activity will not adequately restore strength.)

A client has undergone a right pneumonectomy. The nurse positioning this client following admission from the postanesthesia care unit avoids placing the client in which harmful position? 1.Right lateral 2.Low-Fowler's position 3.Semi-Fowler's position 4.High-Fowler's position

1 Rationale: The client who has a pneumonectomy should not be positioned in the extreme lateral position. This could cause mediastinal shift and compression of remaining lung tissue. The other positions do not pose this risk for the client.)

The nurse is assisting in preparing a list of instructions for an adult client who is being discharged following a tonsillectomy. Which instructions should the nurse include in the list? *Select all that apply.* 1.Avoid hot fluids. 2.Avoid rough foods. 3.Consume milk products. 4.Rest for the next 24 hours. 5.Consume carbonated beverages. 6.Eat ice cream to soothe the throat.

1, 2, 4 (Rationale: Following tonsillectomy, the client is instructed to advance the diet from cool clear liquids to full liquids. Hot fluids and carbonated beverages should be avoided because they may be irritating to the throat. Milk and milk products are avoided because they may cause the client to cough, which can hurt the surgical site. Rough foods and snacks such as raw fruits or vegetables should be avoided for 10 days to protect the scab that forms over the operative site and to prevent bleeding. The client should be instructed to rest in bed or on a couch for 24 hours after the surgical procedure and gradually resume full activity.)

The nurse is caring for a client with laryngitis. Which interventions should the nurse implement? *Select all that apply.* 1.Discourage smoking. 2.Use a room humidifier. 3.Speak only in whispers. 4.Use the intercom to contact the nurse. 5.Use lozenges that contain a topical anesthetic agent.

1, 2, 5 (Rationale: Smoking irritates the throat, so the client is discouraged from smoking. A humidifier will prevent a dry nose and throat. Lozenges with a topical anesthetic agent will decrease throat discomfort. Voice rest means not talking at all, even whispering. There should be a sign on the intercom indicating voice rest and going to the client's room.)

Which diagnostic tests indicate active tuberculosis? *Select all that apply.* 1.Chest x-ray 2.Tuberculin skin test 3.Gastric analysis washings 4.Sputum smear and culture 5.Interferon gamma release assays (IGRA)

1, 3, 4 (Rationale: Active tuberculosis is diagnosed by a chest x-ray, sputum smear, and sputum culture. A diagnosis of active TB is established when the tubercle bacillus has been found in the sputum or gastric washings. Interferon gamma release assays (IGRA) is a diagnostic aid that measures a component of cell-mediated immune reactivity to M. tuberculosis much like the tuberculin skin testing. These test results indicate a need for further evaluation.)

The nurse is preparing to assist a primary health care provider with the insertion of a chest tube. The nurse anticipates that which supplies will be required for the chest tube insertion site? *Select all that apply.* 1.Elastoplast tape 2.Sterile Kerlix dressing 3.Sterile 4 × 4 gauze pads 4.Povidone-iodine solution 5.Petrolatum (Vaseline) gauze

1, 3, 4, 5 (Rationale: The first layer of the chest tube dressing is petrolatum gauze, which allows for an occlusive seal at the chest tube insertion site. Additional layers of sterile 4 × 4 gauze cover this layer, and the dressing is secured with a strong adhesive tape or Elastoplast tape. Povidone-iodine solution may be used to clean the insertion site before the insertion of the chest tube. Kerlix dressing, which is a wrap-type dressing used to wrap and hold dressings in place is not used on the chest; these dressing types are used commonly to wrap dressings placed on the arms or legs.)

The nurse assessing a client diagnosed with laryngeal cancer would note which signs and symptoms? * Select all that apply.* 1.Hemoptysis 2.Kussmaul respirations 3.Enlarged thyroid gland 4.A sensation of a "lump" in the throat 5.Hoarseness lasting more than 3 weeks

1, 4, 5 (Rationale: Hemoptysis, a sensation of a lump in the throat, and hoarseness lasting more than 3 weeks are common signs and symptoms of laryngeal cancer.)

A client is admitted to the emergency department with carbon monoxide poisoning. Which signs and symptoms indicate carbon monoxide poisoning? *Select all that apply.* 1.Mental changes 2.Cardiac irregularities 3.Cherry-red skin color 4.Abnormal arterial blood gas results 5.Negative carboxyhemoglobin levels

1,2,3 (Rationale: Carbon monoxide poisoning results from a buildup of carboxyhemoglobin. Evaluate for early signs and symptoms of carbon monoxide poisoning, which include headache and shortness of breath with mild exertion. Dizziness, nausea, vomiting, and mental changes appear next. As the amount of carbon monoxide in the bloodstream rises, the victim loses consciousness and develops cardiac and respiratory irregularities. A victim usually dies when the carbon monoxide bound with hemoglobin exceeds 70%. Although a cherry-red skin color is a clear indicator of carbon monoxide poisoning, skin color is often found to be pale or bluish with reddish mucous membranes. In carbon monoxide poisoning, the readings of pulse oximetry and the values of arterial blood gases can appear normal despite significant toxic exposure.)

The nurse is collecting data on a client with chronic sinusitis. Which are signs and symptoms of chronic sinusitis? *Select all that apply.* 1.Loss of smell 2.Chronic cough 3.Nasal stuffiness 4.Clear nasal discharge 5.Severe evening headache

1,2,3 (Rationale: Chronic sinusitis is characterized by persistent purulent nasal discharge, a chronic cough caused by nasal discharge, anosmia (loss of smell), nasal stuffiness, and headache that is worse on arising after sleep.)

A clinic nurse is assisting in caring for a client whose chief complaint is the presence of flulike symptoms. Which recommendations by the nurse are therapeutic? *Select all that apply.* 1.Get plenty of rest. 2.Take antipyretics for fever. 3.Increase intake of liquids. 4.Get a flu vaccine immediately. 5.Eat carbohydrates only for energy.

1,2,3 (Rationale: Immunization against influenza is a prophylactic measure and is not used to treat flu symptoms. Treatment for the flu includes getting rest, drinking fluids, and taking in nutritious foods and beverages. Medications such as antipyretics and analgesics also may be used for symptom management. Carbohydrates are not necessarily more important than other elements of a healthy diet.)

The nurse is determining the need for suctioning in a client with an endotracheal tube (ETT) attached to a mechanical ventilator. Which observations are consistent with the need for suctioning? *Select all that apply.* 1.Restlessness 2.Gurgling sounds with respiration 3.Presence of congestion in the lungs 4.Increased pulse and respiratory rates 5.Low peak inspiratory pressure on the ventilator

1,2,3,4 (Rationale: Indications for suctioning include moist, wet respirations; restlessness; congestion on auscultation of the lungs; visible mucus bubbling in the ETT; increased pulse and respiratory rates; and increased peak inspiratory pressures on the ventilator. A low peak inspiratory pressure would indicate a leak in the mechanical ventilation system.)

The nurse is reinforcing instructions to a client following a total laryngectomy about caring for the stoma. Which instructions should the nurse provide to the client? *Select all that apply.* 1.Protect the stoma from water. 2.Soaps should be avoided near the stoma. 3.Wash the stoma daily using a washcloth. 4.Use diluted alcohol on the stoma to clean it. 5.Apply a thin layer of petroleum jelly to the skin surrounding the stoma. 6.Use soft tissues to clean any secretions that accumulate around the stoma.

1,2,3,5 (Rationale: The client with a stoma should be instructed to wash the stoma daily with a washcloth. Soaps, cotton swabs, or tissues should be avoided because their particles may enter and obstruct the airway. The client should be instructed to avoid applying alcohol to a stoma because it is both drying and irritating. A thin layer of petroleum jelly applied to the skin around the stoma helps prevent cracking. The client is instructed to protect the stoma from water.)

The nurse is assisting in caring for a client with a newly inserted tracheostomy. The nurse notes documentation of an airway problem because of thick respiratory secretions. The nurse should monitor for which item as the *best* indicator of an adequate respiratory status? 1.Oxygen saturation of 89% 2.Respiratory rate of 18 breaths per minute 3.Moderate amounts of tracheobronchial secretions 4.Small to moderate amounts of frank blood suctioned from the tube

2 (Rationale: An airway problem could occur following tracheostomy from excessive secretions, bleeding into the trachea, restricted lung expansion caused by immobility, or concurrent respiratory conditions. The respiratory rate of 18 breaths per minute is well within the normal range of 14 to 20 breaths per minute. An oxygen saturation of 89% is less than optimal.)

A client with an oral endotracheal tube attached to a mechanical ventilator is about to begin the weaning process. The nurse asks the primary health care provider whether this process should be delayed temporarily based on administration of which medication to the client in the last hour? 1.Digoxin 2.Lorazepam 3.Furosemide 4.Metoclopramide

2 (Rationale: Antianxiety medications (such as lorazepam) and opioid analgesics are used cautiously or withheld whenever possible in the client being weaned from a mechanical ventilator. These medications may interfere with the weaning process by suppressing the respiratory drive. The other medications do not interfere with the respiratory drive and will not affect the weaning process.)

A client is admitted to the nursing unit following a lobectomy. The nurse caring for the client notes that, in the first hour after admission, the chest tube drainage was 75 mL. During the second hour, the drainage dropped to 5 mL. Which finding does this indicate? 1.This is normal. 2.The tube may be occluded. 3.The lung has fully reexpanded. 4.The client needs to cough and deep breathe.

2 (Rationale: Chest tube drainage in the first 24 hours following thoracic surgery may total 500 to 1000 mL. The sudden drop in drainage between the first and second hour indicates that the tube is possibly occluded and requires further evaluation. Options 1, 3, and 4 are incorrect interpretations.)

The nurse is suctioning a client through a tracheal tube. During the suctioning procedure, the nurse notes on the cardiac monitor that the heart rate has dropped 10 beats. Which should be the nurse's *next* action? 1.Notify the registered nurse immediately. 2.Stop the procedure and oxygenate the client. 3.Continue to suction the client at a quicker pace. 4.Ensure that the suction is limited to 15 seconds.

2 (Rationale: During suctioning the nurse should monitor the client closely for complications including hypoxemia, drop in heart rate due to vagal stimulation, mucosal trauma, hypotension, and paroxysmal coughing. If complications develop, especially cardiac irregularities, the nurse should stop the procedure and oxygenate the client.)

A client is returned to the nursing unit following thoracic surgery with chest tubes in place. During the first few hours postoperatively, the nurse assisting in caring for the client checks for drainage. Which type of drainage is expected? 1.Serous 2.Bloody 3.Serosanguineous 4.Bloody with several clots

2 (Rationale: In the first few hours after surgery, the drainage from the chest tube is bloody. After several hours, it becomes serosanguineous. The client should not experience significant clotting. Proper chest tube function should allow for drainage of blood before it has the chance to clot in the chest or the tubing.)

The nurse reinforces instructing a client how to use an incentive spirometer. Which observation would indicate the ineffective use of this equipment by the client? 1.The client inhales slowly. 2.The client is breathing through the nose. 3.The client removes the mouthpiece from the mouth to exhale. 4.The client forms a tight seal around the mouthpiece with the lips.

2 (Rationale: Incentive spirometry is not effective if the client breathes through the nose. The client should exhale, form a tight seal around the mouthpiece, inhale slowly, hold to the count of 3, and remove the mouthpiece to exhale. The client should repeat the exercise approximately 10 times every hour for best results.)

A client with chronic obstructive pulmonary disease (COPD) who is beginning oxygen therapy asks the nurse why the flow rate cannot be increased. The nurse explains that this can be harmful because it could cause which difficulty? 1.It could be drying to nasal passages. 2.It could decrease the client's oxygen-based respiratory drive. 3.It could increase the risk of pneumonia from drier air passages. 4.It could decrease the client's carbon dioxide-based respiratory drive.

2 (Rationale: Normally, respiratory rate varies with the amount of carbon dioxide present in the blood. In clients with COPD this natural center becomes ineffective after exposure to high carbon dioxide levels for prolonged periods. Instead, the level of oxygen provides the respiratory stimulus. The client with COPD usually cannot increase oxygen levels independently because it could deplete the respiratory drive and lead to respiratory failure. Physician prescriptions are always followed.)

The nurse is reviewing the health care record of a client with a new onset of pleurisy. The nurse notes documentation that the client does not have a pleural friction rub that was auscultated the previous day. How should this finding be interpreted? 1.The medication therapy has been effective. 2.Pleural fluid has accumulated in the inflamed area. 3.The deep breaths that the client is taking are helping. 4.There is a decreased inflammatory reaction at the site.

2 (Rationale: Pleural friction rub is auscultated early in the course of pleurisy before pleural fluid accumulates. Once fluid accumulates in the inflamed area, there is less friction between the visceral and parietal lung surfaces, and the pleural friction rub disappears. Options 1, 3, and 4 are incorrect interpretations.)

The nurse is observing a client with chronic obstructive pulmonary disease (COPD) performing the pursed-lip breathing technique. Which observation by the nurse would indicate accurate performance of this breathing technique? 1.The client's inhalation is twice as long as exhalation. 2.The client's exhalation is twice as long as inhalation. 3.The client loosens the abdominal muscles while breathing out. 4.The client inhales with pursed lips and exhales with the mouth open wide.

2 (Rationale: Prolonging the time for exhaling reduces air trapping because of airway narrowing or collapse in chronic obstructive pulmonary disease. Tightening the abdominal muscles aids in expelling air. Exhaling through pursed lips increases the intraluminal pressure and prevents the airway from collapsing.)

A client enters the urgent care center with epistaxis but no obvious facial injury. The nurse should take which action? 1.Prepare a nasal balloon for insertion. 2.Have the client sit down, lean forward, and apply pressure to the nose. 3.Place the client in the semi-Fowler's position, and apply ice packs to the nose. 4.Position the client in a sitting position, and ask the client to bite on a tongue blade.

2 (Rationale: Sitting the client with the head forward and with pressure applied to the nose is the most effective way to initially control bleeding. Treatment is always directed at a conservative measure first. Placing the client in the semi-Fowler's position causes the client to swallow blood. Preparing a nasal balloon for insertion is invasive and used only when all other efforts have failed. Biting on a tongue blade does not cause cessation of nasal bleeding.)

A client diagnosed with tuberculosis (TB) is distressed over the loss of physical stamina and fatigue. The nurse should provide which explanation for these symptoms? 1.Expected and will last for at least a year 2.Expected and the client should very gradually increase activity as tolerated 3.An unexpected finding with TB, but it should resolve within about 1 month 4.A short-lived problem that should be gone within 1 week of medication therapy

2 (Rationale: The client with TB has significant fatigue and loss of physical stamina. This can be very frightening for the client. The nurse teaches the client that this will resolve as the therapy progresses and that the client should gradually increase activity as energy levels permit.)

The nurse determines that which client is at greatest risk for development of acute respiratory distress syndrome (ARDS)? 1.A client with blunt chest trauma 2.A client with pancreatitis and gram-negative sepsis 3.A client who has received 1 unit of packed red blood cells 4.A client with acute pulmonary edema after myocardial infarction

2 (Rationale: The client with pancreatitis and gram-negative sepsis is at greatest risk of developing ARDS because of the presence of two risk factors for its development. Although the client with blunt chest trauma is also at risk, those who have multiple risk factors have a three to four times greater incidence for development of ARDS. Massive blood transfusion is a risk factor for ARDS; however, this client has received only 1 unit. Pulmonary edema after myocardial infarction occurs when increased pulmonary capillary hydrostatic pressure causes flooding of the pulmonary interstitial spaces and then the alveoli. The pulmonary edema that occurs in ARDS is due to damage to pulmonary vasculature resulting in increased pulmonary capillary permeability.)

The nurse is reviewing the record of a client with acute respiratory distress syndrome (ARDS). The nurse determines that which finding documented in the client's record is consistent with the *most* expected characteristic of this disorder? 1.Central cyanosis 2.Arterial Pao2 of 48 3.Arterial Pao2 of 81 4.Respiratory rate of 10 breaths per minute

2 (Rationale: The most characteristic sign of ARDS is increasing hypoxemia with a Pao2 of less than 60 mm Hg. This occurs despite increasing levels of oxygen that are administered to the client. The client's earliest sign is an increased respiratory rate. Breathing then becomes labored, and the client may exhibit air hunger, retractions, and peripheral cyanosis.)

The nurse has finished suctioning a client. The nurse should use which parameters to *best* determine the *effectiveness* of suctioning? 1.Client skin color (pink) 2.Breath sounds are clear 3.Client statement of comfort 4.Sao2 is 98% by pulse oximetry

2 (Rationale: The nurse evaluates the effectiveness of the suctioning procedure by auscultating breath sounds. This helps determine if the respiratory tract is clear of secretions. In addition, breath sounds must be auscultated before every suctioning procedure. Options 1, 3, and 4 do not determine the effectiveness of suctioning.)

The nurse is assisting a client who underwent radical neck surgery to get out of bed. How does the nurse provide support to this client who is afraid to move the head? 1.The nurse applies a soft cervical collar. 2.The nurse places a hand behind the client's head. 3.The nurse raises the head of the bed 90 degrees. 4.The nurse assists the client to roll to the side of the bed and sit up slowly.

2 (Rationale: The nurse provides the most support to the surgical site by placing a hand behind the client's head. Options 3 and 4 involve little assistance or support by the nurse. Option 1 is unnecessary and could occlude a tracheostomy if one is in place.)

A client reports the chronic use of nasal sprays. The nurse reinforces instructions to this client about which piece of information related to chronic use of nasal sprays? 1.Nosebleeds are common. 2.The protective mechanism of the nose may be damaged. 3.It is acceptable to double the dose if one dose is ineffective. 4.Fungal infections of the nose may occur because of container contamination.

2 (Rationale: The protective mechanisms of the nose may be altered with the chronic use of nasal sprays. Fungal infections occur with oral inhalers but not nasal sprays. Nosebleeds are uncommon. The client should not double-dose medications to increase their effect.)

The nurse is planning care for a client whose oxygenation is being monitored by a pulse oximeter. Which intervention is important to ensure accurate monitoring of the client's oxygenation status? 1.Tape the sensor to the client's finger. 2.Instruct the client not to move the sensor. 3.Place the sensor on a finger below the blood pressure cuff. 4.Notify the primary health care provider immediately of an O2 saturation less than 90%.

2 (Rationale: The pulse oximeter passes a beam of light through the tissue, and a sensor attached to the fingertip, toe, or earlobe measures the amount of light absorbed by the oxygen-saturated hemoglobin. The oximeter then gives a reading of the percentage of hemoglobin that is saturated with oxygen (Sao2). Motion at the sensor site changes light absorption. The motion mimics the pulsatile motion of blood, and because the detector cannot distinguish between movement of blood and movement of the finger, results can be inaccurate. The sensor should not be placed distal to blood pressure cuffs, pressure dressings, arterial lines, or any invasive catheters. The sensor should not be taped to the client's finger. If values fall below preset norms (usually 90%), the client should be instructed to deep breathe if this is appropriate.)

The nurse is admitting a client with a possible diagnosis of chronic bronchitis. The nurse collects data from the client and notes that which signs/symptoms support this diagnosis? *Select all that apply.* 1.Scant mucus 2.Early onset cough 3.Marked weight loss 4.Purulent mucous production 5.Mild episodes of dyspnea

2,4,5 (Rationale: Key features of pulmonary emphysema include dyspnea that is often marked, late cough (after onset of dyspnea), scant mucous production, and marked weight loss. By contrast, chronic bronchitis is characterized by an early onset of cough (before dyspnea), copious purulent mucous production, minimal weight loss, and milder episodes of dyspnea.)

A client with a diagnosis of lung cancer returns to the nursing unit after a left pneumonectomy. Which nursing actions should be done? *Select all that apply.* 1.Turn completely on the side. 2.Administer humidified oxygen. 3.Instruct on the use of the incentive spirometer. 4.Monitor vital signs and pulse oximetry frequently. 5.Place in respiratory isolation to prevent infection.

2, 3, 4 (Rationale: A client with a pneumonectomy can be turned slightly and supported with a pillow, but complete lateral positioning is contraindicated because of pressure on the bronchial stump or shifting of mediastinal contents. In addition, the surgeon's prescription for positioning is always checked and followed. The client needs to receive oxygen and use an incentive spirometer to prevent atelectasis in the remaining lung. Vital signs and pulse oximetry need to be monitored frequently. The client should not be placed in respiratory isolation to prevent infection; this is unnecessary.)

Which nursing actions would contribute to monitoring and maintaining a patent airway for the postoperative client? *Select all that apply.* 1.Repositioning client every 4 hours 2.Position on the side until fully recovered 3.Encouraging coughing and deep breathing 4.Monitoring pulse oximetry readings frequently 5.Encouraging the use of an incentive spirometer

2, 3, 4, 5 (Rationale: Monitoring and maintaining a patent airway is a nursing responsibility. The nurse should monitor oxygen saturation closely and administer oxygen as prescribed. The use of an incentive spirometer is especially helpful to prevent atelectasis and hypoventilation. Unless contraindicated, the client should be positioned on the side or with the head turned to the side to prevent aspiration until fully recovered, alert, and with the gag reflex intact. The client is encouraged to deep breathe and cough every 2 hours to prevent atelectasis. The client should be repositioned every 2 hours, which changes the distribution of gas and blood flow in the lungs and helps move secretions.)

The nurse is planning therapeutic interventions for a client who experienced a rib fracture 2 days earlier. The nurse understands that which intervention should be included? *Select all that apply.* 1.Ice 2.Rest 3.Local heat 4.Analgesics 5.Oxygen by nasal cannula

2,3,4 (Rationale: Common therapies for fractured ribs include rest, analgesics, and the local application of heat that speeds the resolution of inflammation. Ice is not effective 48 hours after injury, and oxygen may not be necessary. Analgesics that cause respiratory depression are avoided.)

A hospitalized client is dyspneic and has been diagnosed with left pneumothorax by chest x-ray. Which sign or symptom observed by the nurse clearly indicates that the pneumothorax is rapidly worsening? 1.Hypertension 2.Pain with respiration 3.Tracheal deviation to the right 4.Respiratory rate of 18 breaths per minute

3 (Rationale: A pneumothorax is characterized by distended neck veins, displaced point of maximal impulse (PMI), subcutaneous emphysema, tracheal deviation to the unaffected side, decreased fremitus, and worsening cyanosis. The client could have pain with respiration even with a milder pneumothorax. The increased intrathoracic pressure causes the blood pressure to fall, not rise. A respiratory rate of 18 breaths per minute is within the normal range.)

A client had thoracic surgery 2 days ago and has a chest tube in place connected to a closed chest tube system. The nurse notes continuous bubbling in the water seal chamber. The nurse determines which? 1.This is normal on the second postoperative day. 2.The client has a large amount of fluid that is being evacuated by the system. 3.There is a leak in the system that requires immediate investigation and correction. 4.This is due to the suction applied to the system, which is set at 20 cm of suction pressure.

3 (Rationale: Continuous bubbling in the water seal chamber of a chest tube indicates that a leak exists somewhere in the system and air is being sucked into the apparatus. The nurse needs to assess the system and initiate corrective action that may include notifying the primary health care provider. Bubbling may occur intermittently with the evacuation of a pneumothorax, but it should not be continuous, especially with a client who had surgery 2 days earlier. Hemothorax results in accumulation of drainage in the collection chamber but does not cause bubbling in the water seal chamber. Application of suction to the system causes bubbling in the suction control chamber but not the water seal chamber.)

A client who has had a radical neck dissection begins to hemorrhage at the incision site. Which action by the nurse would be contraindicated? 1.Monitoring the client's airway 2.Applying manual pressure over the site 3.Lowering the head of the bed to a flat position 4.Calling the primary health care provider immediately

3 (Rationale: If the client begins to hemorrhage from the surgical site following radical neck dissection, the nurse elevates the head of the bed to maintain airway patency and prevent aspiration. The nurse applies pressure over the bleeding site, contacts the registered nurse immediately who will then call the primary health care provider immediately.)

A client has undergone fluoroscopy-assisted aspiration biopsy of a lung lesion. The nurse determines that the client is experiencing complications from the procedure if the nurse makes which observation? 1.Skin pink, warm, and dry 2.Oxygen saturation 97% by pulse oximetry 3.Absence of breath sounds in the right upper lobe 4.Pulse rate of 80 beats per minute, up from 74 beats per minute

3 (Rationale: Pneumothorax and bleeding are possible complications of this procedure. The client is observed for signs of respiratory difficulty such as dyspnea, change in breath sounds, vital signs, pallor, and diaphoresis. Observation of the sputum for traces of blood or hemoptysis also is indicated. The absence of breath sounds in the right upper lobe indicates a potential pneumothorax.)

The nurse is monitoring a client following a motor vehicle crash. Which finding would indicate a need for chest tube placement? 1.Chest pain and shortness of breath 2.Peripheral cyanosis and hypotension 3.Shortness of breath and tracheal deviation 4.Decreasing oxygen saturation and bradypnea

3 (Rationale: Shortness of breath and tracheal deviation result when lung tissue and alveoli have collapsed. Air entering the pleural cavity causes the lung to lose its normal negative pressure. The increasing pressure in the affected side displaces contents to the unaffected side. Shortness of breath results from decreased area available for diffusion of gases. Chest pain and shortness of breath are more commonly associated with myocardial ischemia or infarction. Clients requiring chest tubes exhibit decreasing oxygen saturation but will more likely experience tachypnea related to the hypoxia. Peripheral cyanosis is caused by circulatory disorders. Hypotension may be a result of tracheal deviation and impedance of venous return to the heart. It may also be the result of other problems such as a failing heart.)

The nurse is collecting data from a client with pneumonia. Chest auscultation over areas of consolidation reveals this breath sound. (Refer to audio.) The nurse should interpret this sound to be indicative of which breath sound? 1.Pleural friction rub 2.Vesicular breath sounds 3.Bronchial breath sounds 4.Bronchovesicular breath sounds

3 (Rationale: The sound that the nurse hears is a bronchial breath sound. Bronchial breath sounds are loud, high-pitched sounds that resemble air blowing through a hollow pipe. The expiration phase is louder and longer than the inspiration phase, and there is a distinct pause between the inspiration and expiration phase. Bronchial breath sounds normally are heard only over the manubrium. When they are heard over the periphery of the lung, they indicate abnormal sound transmission because of consolidation of lung tissue as in pneumonia. A pleural friction rub is a superficial, low-pitched, coarse rubbing or grating sound that sounds like two surfaces rubbing together and is heard in the client with pleurisy. Vesicular breath sounds normally are heard over the lesser bronchi, bronchioles, and lobes (peripheral lung fields). These sounds are soft and low-pitched and resemble a sighing or gentle rustling. Bronchovesicular breath sounds normally are heard over the first and second intercostal spaces at the sternal border anteriorly and at T4 medial to the scapula posteriorly. These sounds are a mixture of bronchial and vesicular breath sounds and are moderately pitched with a medium intensity.)

A client with right pleural effusion by chest x-ray is being prepared for a thoracentesis. The client experiences dizziness when sitting upright. The nurse assists the client to which position for the procedure? 1.Sims' position with the head of the bed flat 2.Prone with the head turned to the side supported by a pillow 3.Left side-lying with the head of the bed elevated at 45 degrees 4.Right side-lying with the head of the bed elevated at 45 degrees

3 (Rationale: To facilitate removal of fluid from the chest wall, two positions may be used. The client may be positioned sitting on the edge of the bed, leaning over the bedside table with his or her feet supported on a stool. The other position is lying in bed on the unaffected side with the head of the bed elevated 45 degrees (Fowler's position). The other options are incorrect because they do not facilitate drainage of fluid to an area where it can be easily removed with thoracentesis.)

The nurse is observing a nursing student listening to the breath sounds of a client. The nurse intervenes if the student performs which incorrect procedure? 1.Asks the client to sit upright 2.Uses the diaphragm of the stethoscope 3.Places the stethoscope on the client's gown 4.Asks the client to breathe slowly and deeply through the mouth

3 (Rationale: To listen to breath sounds, the stethoscope always is placed directly on the client's skin, and not over a gown or clothing. The nurse asks the client to sit up and breathe slowly and deeply through the mouth. Breath sounds are auscultated using the diaphragm of the stethoscope, which is warmed before use.)

The nurse is performing nasotracheal suctioning of a client. The nurse determines that the client is adequately tolerating the procedure if which observation is made? 1.Skin color becomes cyanotic. 2.Secretions are becoming bloody. 3.Coughing occurs with suctioning. 4.Heart rate decreases from 78 to 54 beats per minute.

3 Rationale: The nurse monitors for adverse effects of suctioning, which include cyanosis, excessively rapid or slow heart rate, or the sudden development of bloody secretions. If they occur, the nurse stops suctioning and reports these signs to the primary health care provider immediately. Coughing is a normal response to suctioning for the client with an intact cough reflex and does not indicate that the client cannot tolerate the procedure.

The nurse is assessing a client diagnosed with sinusitis. Which are signs and symptoms of sinusitis? *Select all that apply.* 1.A high fever 2.Nuchal rigidity 3.Headache, especially in the morning 4.Elevated white blood cell (WBC) count 5.Feeling of heaviness over affected areas

3, 4, 5 (Rationale: Signs and symptoms of sinusitis include a feeling of heaviness over the affected areas. This can feel like a toothache if maxillary sinusitis or a headache, especially in the morning, for frontal sinusitis. Nasal drainage can become purulent. The white blood count is elevated. A high fever and nuchal rigidity are signs and symptoms of meningitis, which is a possible complication of sinusitis.)

The nurse is monitoring the respiratory status of a client following insertion of a tracheostomy. The nurse understands that oxygen saturation measurements obtained by pulse oximetry may be inaccurate if the client has which coexisting problems? *Select all that apply.* 1.Fever 2.Epilepsy 3.Hypotension 4.Respiratory failure 5.Use of peripheral vasoconstrictors

3, 5 (Rationale: Hypotension, shock, or the use of peripheral vasoconstricting medications may result in inaccurate pulse oximetry readings from impaired peripheral perfusion. Fever and epilepsy would not affect the accuracy of measurement. Respiratory failure also would not affect the accuracy of measurement, although the readings may be abnormally low.)

The nurse is caring for a client with tuberculosis (TB) who is fearful of the disease and anxious about the prognosis. In planning nursing care, the nurse should incorporate which action as the *best* strategy to assist the client in coping with the disease? 1.Ask family members if they wish a psychiatric consult. 2.Allow the client to deal with the disease in an individual fashion. 3.Encourage the client to visit with the pastoral care department chaplain. 4.Provide reassurance that continued compliance with medication therapy is the most proactive way to cope with the disease.

4 (Rationale: A primary role of the nurse in working with the client with tuberculosis is to teach the client about medication therapy. The anxious client may not absorb information optimally. The nurse continues to reinforce teaching using a variety of methods (repetition, teaching aids) and teaches the family about the medications as well. The most effective way of coping with the disease is to learn about the therapy that will eradicate it. This gives the client a measure of power over the situation and outcome.)

While assessing a client who is admitted to the hospital with a diagnosis of pleurisy, the nurse would note which characteristic symptom of this disorder? 1.Early morning fatigue 2.Dyspnea that is relieved by lying flat 3.Pain that worsens when the breath is held 4.Knifelike pain that worsens on inspiration

4 (Rationale: A typical symptom with pleurisy is a knife like pain that worsens on inspiration. This is a result of the friction caused by the rubbing together of inflamed pleural surfaces. This pain usually disappears when the breath is held because these surfaces stop moving. The client does not experience early morning fatigue or dyspnea relieved by lying flat.)

The primary health care provider has prescribed amantadine for a client admitted to the hospital for hip replacement surgery. The nurse recognizes that this medication was prescribed because the client's history showed recent exposure to which? 1.Bronchitis 2.Pneumonia 3.Tuberculosis 4.Type A influenza

4 (Rationale: Antiviral medications may be used in specific client populations. If a person is known to be at high risk for influenza and has been exposed to type A influenza, the provider may choose to provide prophylaxis with an antiviral agent such as amantadine. Amantadine would not be prescribed to prevent bronchitis, pneumonia or tuberculosis.)

The nurse is providing endotracheal suctioning to a client who is mechanically ventilated when the client becomes restless and tachycardic. Which action should the nurse take? 1.Call the primary health care provider. 2.Contact the respiratory department to suction the client. 3.Hyperoxygenate and hyperventilate the client with an Ambu bag and resuction. 4.Monitor vital signs and discontinue attempts at suctioning until the client is stabilized.

4 (Rationale: If a client becomes cyanotic or restless or develops tachycardia, bradycardia, or another abnormal heart rhythm while suctioning, the nurse must discontinue suctioning attempts until the client is stabilized. It is also important to monitor vital signs and pulse oximetry and preoxygenate the client for any repeated suctioning attempts. If the client's condition continues to deteriorate, then the respiratory department and primary health care provider may need to be notified.)

A client with a nasal tumor is being admitted to the hospital. The nurse collects data about which *primary* symptom that the client is expected to exhibit? 1.Epistaxis 2.Headache 3.Runny nose 4.Nasal obstruction

4 (Rationale: Nasal obstruction is the most common symptom associated with a nasal tumor because the tumor occupies space in the nasal area. Bleeding (epistaxis) may occur but is not a primary sign. Headache and a runny nose are not compatible with the clinical picture of a client with a nasal tumor.)

A client with a respiratory disorder has anorexia secondary to fatigue and dyspnea while eating. The nurse determines that the client has followed the recommendations to improve intake if which action is taken? 1.The client selects foods that are very dry. 2.The client increases the use of milk products. 3.The client increases the use of stimulants such as caffeine. 4.The client plans to eat the largest meal of the day at a time when hungry.

4 (Rationale: The client is taught to plan the largest meal of the day at a time when the client is most likely to be hungry. It is also beneficial to eat four to six small meals per day if needed. The client avoids dry foods, which are hard to chew and swallow. The client also avoids milk and chocolate, which have a tendency to thicken saliva and secretions. Finally, the client should avoid the use of caffeine, which contributes to dehydration by promoting diuresis.)

The nurse is assisting a primary health care provider with the insertion of an endotracheal tube (ETT). The nurse should plan which as a final measure to determine correct tube placement? 1.Hyperoxygenate the client. 2.Tape the tube securely in place. 3.Listen for bilateral breath sounds. 4.Verify placement by a chest x-ray.

4 (Rationale: The final measure to determine ETT placement is to verify it by a chest x-ray. The chest x-ray shows the exact placement of the tube in the trachea, which should be above the bifurcation of the right and left mainstream bronchi. The other options are incorrect because they are completed initially after tube placement.)

The nurse is assessing a client with multiple traumas who is at risk for developing acute respiratory distress syndrome (ARDS). The nurse should assess for which *earliest* sign of acute respiratory distress syndrome? 1.Diffuse crackles 2.Bilateral wheezing 3.Intercostal retractions 4.Increased respiratory rate

4 (Rationale: The first sign of ARDS is usually increased respiratory rate. Auscultation of the lungs may reveal fine crackles. The client may be restless, agitated, and confused. The pulse rate increases and a cough may be present. These early signs are followed by progressively worsening dyspnea with retractions, cyanosis, and diaphoresis. Diffuse crackles and rhonchi may be heard on auscultation)

The nurse is caring for a client who is anxious and is experiencing dyspnea and restlessness from hypoxemia associated with pulmonary edema. Auscultation of the lungs reveals these breath sounds. (Refer to audio.) The nurse determines that these breath sounds usually result from which cause? 1.Obstruction of the bronchus 2.Inflammation of the pleural surfaces 3.Passage of air through a narrowed airway 4.Opening of small airways that contain fluid

4 (Rationale: The sounds that the nurse hears are high-pitched crackles. Crackles are audible when there is a sudden opening of small airways that contain fluid. Crackles are usually heard during inspiration, and do not clear with a cough. They resemble the sound of a lock of hair being rubbed between the thumb and forefinger and are heard in conditions such as pulmonary edema. High-pitched crackles are characteristically fine and are high-pitched discontinuous popping noises (nonmusical sounds) heard during the end of inspiration. Medium-pitched crackles produce a moist sound about halfway through inspiration. Coarse crackles are low-pitched bubbling sounds that start early in inspiration and extend into the first part of expiration. Rhonchi (low-pitched, coarse, loud, low snoring or moaning sounds) are heard in conditions causing obstruction of the bronchus or trachea. A pleural friction rub (a superficial low-pitched coarse rubbing or grating sound) is heard when the pleural surfaces are inflamed. Passage of air through a narrowed airway is associated with wheezes (a high-pitched musical sound similar to a squeak))

A primary health care provider is about to remove a chest tube from a client. Once the dressing is removed and the sutures have been cut, the nurse assisting the primary health care provider asks the client to do which action? 1.Exhale immediately. 2.Breathe in and out rapidly. 3.Breathe deeply and rapidly. 4.Perform the Valsalva maneuver.

4 (Rationale: When the chest tube is removed, the client is asked to perform the Valsalva maneuver (take a deep breath, exhale, and bear down). The tube is then quickly withdrawn, and an airtight dressing is taped in place. The pleura seals itself off and the wound heals in less than a week. Therefore, options 1, 2, and 3 are incorrect.)

A client who is experiencing severe respiratory acidosis has a potassium level of 6.2 mEq/L. The nurse determines this result is *best* characterized by which interpretation? 1.Expected and indicates the result of massive hemolysis 2.Unexpected and indicates a concurrent history of renal insufficiency 3.Unexpected and indicates a deficit of hydrogen ions in the bloodstream 4.Expected and indicates that acidosis has driven hydrogen ions into the cell, forcing potassium out

4 (Rationale: With severe respiratory acidosis, compensatory mechanisms fail. As hydrogen ion concentrations continue to rise, they are driven into the cell forcing intracellular potassium out. This is an expected finding in this situation. Options 1, 2, and 3 are incorrect interpretations.)

The nurse is preparing to perform nasotracheal suctioning on a client. The nurse places the client's bed in which position to *effectively* perform this procedure? Refer to figure. a) Fowlers b) Trendelenburg's c) reverse Trendelenburg's d) flat position

a) (Rationale: The client is placed in the Fowler's position for nasotracheal suctioning. This position promotes lung expansion and is also the preferred position for eating and nasogastric tube insertion. Option 2 identifies Trendelenburg's position. This position is used for postural drainage and facilitates venous return in clients with poor peripheral circulation. Option 3 identifies reverse Trendelenburg's position. This position promotes gastric emptying and prevents esophageal reflux. Option 4 identifies a flat position that is used for clients with vertebral injuries and for clients in cervical traction. A flat position also may be used for clients who are hypotensive.)

The nurse is assigned to care for a client after a left pneumonectomy. Which position is contraindicated for this client? 1.Lateral position 2.Low-Fowler's position 3.Semi-Fowler's position 4.Head of the bed elevation at 40 degrees

1 (Rationale: Complete lateral positioning is contraindicated for a client following pneumonectomy. Because the mediastinum is no longer held in place on both sides by lung tissue, lateral positioning may cause mediastinal shift and compression of the remaining lung. The head of the bed should be elevated.)

A client who is postoperative with incisional pain complains to the nurse about completing respiratory exercises. The client is willing to do the deep breathing exercises but states that it hurts to cough. The nurse provides gentle encouragement and appropriate pain management to the client, knowing that coughing is needed for which reason? 1.To expel mucus from the airways 2.To dilate the terminal bronchioles 3.To exercise the muscles of respiration 4.To provide for increased oxygen tension in the alveoli

1 (Rationale: Coughing is one of the protective reflexes. Its purpose is to move mucus that is in the airways upward toward the mouth and nose. Coughing is needed in the postoperative client to mobilize secretions and expel them from the airways. The other options do not accurately address the purpose of coughing in the postoperative client.)

The nurse is planning to suction a client through a tracheostomy tube. Which is the amount of time for application of suction during withdrawal of the catheter? 1.10 seconds 2.25 seconds 3.30 seconds 4.35 seconds

1 (Rationale: During suctioning, the nurse should apply suction during the withdrawal of the catheter for a period of 5 to 10 seconds. Suction applied longer than this can cause hypoxia in the client.)

The nurse is assisting in caring for a client with a tracheal tube attached to a ventilator when an alarm sounds. Which action should the nurse do first? 1.Check the client. 2.Check the ventilator. 3.Manually ventilate the client with a resuscitation bag. 4.Call the respiratory therapist or rapid response team.

1 (Rationale: For a client receiving mechanical ventilation, always check the client first and then check the ventilator. A resuscitation bag should be available at the bedside for all clients receiving mechanical ventilation. If the cause of the alarm cannot be determined, ventilate the client manually with a resuscitation bag until the problem is corrected. The nurse needs to determine if the respiratory therapist or rapid response team needs to be called.)

A client with arthritis of the hands and fingers is having difficulty using a metered-dose inhaler (MDI). The nurse suggests asking the primary health care provider for which prescription? 1.Use of a spacer 2.Use of a nebulizer 3.Use of an oral (pill) form of the medication 4.Use of an intravenous (IV) form of the medication

1 (Rationale: For a client with arthritis or other conditions that limit the use of the hands, the use of a spacer may prove beneficial. A spacer allows the medication to be delivered deep into pulmonary tissues even if the client has difficulty with coordination. MDIs may be difficult to use because they require coordination and adequate hand motion to hold the canister at the proper distance (1½ to 2 inches from the mouth), depress the canister, and inhale. A spacer is especially useful for clients who are older or who have difficulty using an MDI. The other options are incorrect.)

A client with acquired immunodeficiency syndrome (AIDS) has become infected with histoplasmosis. The nurse monitors the client for which manifestation of histoplasmosis? 1.Dyspnea 2.Headache 3.Weight gain 4.Hypothermia

1 (Rationale: Histoplasmosis is an opportunistic fungal infection that can occur in the client with AIDS. The infection begins as a respiratory infection and can progress to disseminated infection. Typical signs and symptoms include fever, dyspnea, cough, and weight loss. There also may be enlargement of the client's lymph nodes, liver, and spleen.)

A client with acquired immunodeficiency syndrome (AIDS) has histoplasmosis. Which sign/symptom should the nurse expect the client to experience? 1.Dyspnea 2.Headache 3.Weight gain 4.Hypothermia

1 (Rationale: Histoplasmosis is an opportunistic fungal infection that can occur in the client with AIDS. The infection begins as a respiratory infection and can progress to disseminated infection. Typical signs and symptoms include fever, dyspnea, cough, and weight loss. There may be an enlargement of the client's lymph nodes, liver, and spleen as well.)

The nurse is caring for a client with fractured ribs. Which statement indicates a *need for further teaching*? 1."My ribs will be healed in a month." 2."I should only need pain med for a week." 3."I should stay calm and rest after taking pain med." 4."I need to support my ribs when I deep breathe and cough."

1 (Rationale: Rib fractures heal in 6 weeks. Pain typically persists for 5 to 7 days. After medications are given, the client should be provided a calm environment and encouraged to rest. After rib fractures, instruct the client to support the fractured ribs while deep breathing and coughing.)

The nurse is taking the nursing history of a client with silicosis. The nurse checks whether the client wears which item during periods of exposure to silica particles? 1.Mask 2.Gown 3.Gloves 4.Eye protection

1 (Rationale: Silicosis results from chronic, excessive inhalation of particles of free crystalline silica dust. The client should wear a mask to limit inhalation of this substance, which can cause restrictive lung disease after years of exposure. The other options are not necessary.)

The nurse is assisting in caring for a postoperative client who had a pneumonectomy. The nurse monitors the client for which adverse sign/symptom indicating acute pulmonary edema? 1.Frothy sputum 2.Pain with deep breathing 3.Increased chest tube drainage 4.Respiratory rate of 20 breaths per minute

1 (Rationale: The client developing pulmonary edema after pneumonectomy demonstrates dyspnea, cough, frothy sputum, crackles, and possibly cyanosis. A respiratory rate of 20 breaths per minute is within normal limits. Pain with deep breathing is expected and managed with analgesics. The client with pneumonectomy usually does not have a chest tube.)

A clinic nurse is reinforcing instructions to a client with a diagnosis of pharyngitis. Which intervention should the client be encouraged to perform? 1.Avoid foods that are highly seasoned. 2.Restrict fluid intake to 1000 mL daily. 3.Drink warm herbal tea throughout the day. 4.Substitute hot chocolate in place of coffee.

1 (Rationale: The client with pharyngitis should be instructed to consume cool clear fluids, ice chips, or ice pops to soothe the painful throat. Milk and milk products are avoided because they tend to increase mucous production. Foods that are highly seasoned are irritating to the throat and should be avoided, and the client should be instructed to drink 2000 to 3000 mL of fluid daily unless contraindicated.)

The nurse is preparing to perform chest physiotherapy (CPT) on a client. In performing postural drainage, which statement is incorrect? 1.Breathe in a fast-paced pattern. 2.Perform postural drainage before meals. 3.Perform good mouth care after the procedure. 4.Instruct client not to sit up between position changes.

1 (Rationale: The goal of chest physiotherapy is to mobilize secretions for improved respiratory function. The nurse must determine which areas of the lungs should be targeted for this technique. The client should be instructed to breathe slowly and deeply throughout the procedure. The client should not sit up between position changes. Perform postural drainage before meals or tube feedings. It may be ordered after respiratory treatments with bronchodilators. After postural drainage is completed, good mouth care—including brushing the teeth and using a refreshing mouthwash—should be performed.)

The nurse notes that a hospitalized client has experienced a positive reaction to the tuberculin skin test. Which action by the nurse is priority? 1.Report the findings. 2.Document the finding in the client's record. 3.Call the employee health service department. 4.Call the radiology department for a chest x-ray.

1 (Rationale: The nurse who interprets a tuberculin skin test as positive notifies the PHCP immediately. The PHCP would prescribe a chest x-ray to determine whether the client has clinically active tuberculosis or old healed lesions. A sputum culture would be done to confirm the diagnosis of active tuberculosis. The client is placed on tuberculosis precautions prophylactically until a final diagnosis is made. The findings are documented in the client's record, but this action is not the highest priority. Calling the employee health service would be of no benefit to the client.)

The nurse is assigned to assist with caring for a client who has a chest tube. The nurse notes fluctuations of the fluid level in the water-seal chamber. Based on this observation, which action would be appropriate? 1.Continue to monitor. 2.Empty the drainage. 3.Encourage the client to deep breathe. 4.Encourage the client to hold his or her breath periodically.

1 (Rationale: The presence of fluctuations in the fluid level in the water-seal chamber indicates a patent drainage system. With normal breathing, the water level rises with inspiration and falls with expiration. The apparatus and all connections must remain airtight at all times, and the drainage is never emptied because of the risk of disruption in the closed system, which can result in lung collapse. Encouraging the client to deep breathe is unrelated to this observation. The client is not told to hold his or her (client) breath.)

The nurse is preparing to obtain a sputum specimen from the client. Which nursing action is *essential* in obtaining a proper specimen? 1.Have the client take three deep breaths. 2.Limit fluids before obtaining the specimen. 3.Ask the client to obtain the specimen after eating. 4.Ask the client to spit into the collection container.

1 (Rationale: To obtain a sputum specimen, the client should brush his or her teeth to reduce mouth contamination. The client should then take three deep breaths and cough into a sputum specimen container. The client should be encouraged to cough and not spit so that sputum can be obtained. Sputum can be thinned by fluids or by a respiratory treatment such as inhalation of nebulized saline or water. The optimal time to obtain a specimen is on arising in the morning.)

The nurse is caring for a client with an endotracheal tube attached to a ventilator. The high-pressure alarm sounds on the ventilator. The nurse prepares to perform which priority nursing intervention? 1.Suction the client. 2.Check for a disconnection. 3.Notify the respiratory therapist. 4.Evaluate the tube cuff for a leak.

1 (Rationale: When the high-pressure alarm sounds on a ventilator, it is most likely caused by an obstruction. The obstruction can be caused by the client biting on the tube, kinking of the tubing, or mucous plugging requiring suctioning. It is also important to check the tubing for the presence of any water and determine whether the client is out of rhythm with breathing with the ventilator. A disconnection or a cuff leak can result in the sounding of the low-pressure alarm. The respiratory therapist should be notified if the nurse could not determine the cause of the alarm.)

The nurse is caring for a client following segmental resection of the upper lobe of the left lung. The nurse notes 700 mL of grossly bloody drainage in the chest tube drainage system during the first hour following surgery. Which statement represents the nurse's accurate interpretation of this finding? 1.This finding requires further data collection. 2.This finding indicates the need for autotransfusion. 3.This finding is expected following this type of surgery. 4.This finding indicates a malfunction of the chest tube drainage system.

1 (Rationale: Within the first 2 hours following surgery, 100 to 300 mL of drainage is expected. An amount of 700 mL is excessive and indicates that hemorrhage may be occurring, and the client requires further data collection. The primary health care provider should be notified. Options 2, 3, and 4 are incorrect.)

A client with active tuberculosis (TB) demonstrates less-than-expected interest in learning about the prescribed medication therapy. Which technique would the nurse ultimately need to employ in order to encourage participation? 1.Directly observed therapy 2.More medication instructions 3.Involvement of the family in teaching 4.Reinforcement by the primary health care provider

1 Rationale: Tuberculosis is a highly communicable disease that is reportable to local public health departments. Each of these agencies has regulations that may be enforced to ensure compliance with tuberculosis therapy. The client may be required to have directly observed therapy to reduce the risk to the general public. This involves having a responsible person actually observe the client taking the medication each day.

The nurse is caring for a newly admitted client with pneumonia. The primary health care provider has prescribed a sputum specimen for culture and sensitivity. The nurse should perform the actions concerning the sputum collection in which priority order? Arrange the actions in the order that they should be performed. All options must be used. Send the specimen immediately to the laboratory. Administer the prescribed antibiotic Obtain and label a sterile container. Have the client brush teeth and rinse mouth with water. Have the client take several deep breaths before coughing. Have the client expectorate sputum (not saliva) into sterile container.

1 Obtain and label a sterile container. 2 Have the client brush teeth and rinse mouth with water. 3 Have the client take several deep breaths before coughing. 4 Have the client expectorate sputum (not saliva) into sterile container. 5 Send the specimen immediately to the laboratory. 6 Administer the prescribed antibiotic

A client is being prepared for a thoracentesis. The nurse reinforces instructions with the client given by the registered nurse. Which points should be included in the instructions? *Select all that apply.* 1.The client leans over a bedside table. 2.The client should sit on the edge of the bed. 3.The procedure involves obtaining a biopsy. 4.A time-out is performed before the procedure. 5.The procedure is performed during a bronchoscopy. 6.A local anesthetic is administered before the procedure.

1,2,4,6 (Rationale: A thoracentesis is a procedure in which fluid is removed from the pleural space. The procedure involves insertion of a needle percutaneously and then removal of the fluid by connecting the needle to a vacuum bottle. Before the thoracentesis, the nurse needs to check for allergies because a local anesthetic is administered. A time-out is performed in which the client identification, coagulation studies, and area of the pleural effusion is verified. A chest x-ray is performed after the procedure. A potential complication is a pneumothorax. The client sits on the bedside and leans over a bedside table, which exposes the area between the ribs. A lung biopsy is often done during a bronchoscopy.)

The client is diagnosed with pleurisy. The nurse should expect to see which signs and symptoms? *Select all that apply.* 1.Pleural friction rub 2.Sharp, knife-like pain 3.Cyanosis of lips and nailbeds 4.Pain that occurs on both sides of the chest 5.Pain that occurs most often during inspiration

1,2,5 (Rationale: Pleurisy is inflammation of the pleura. The most characteristic symptom of pleurisy is abrupt and severe pain. The pain almost always occurs on one side of the chest. Pleurisy pain is sharp, knife-like, and abrupt in onset and is most evident during inspiration. This causes shallow breathing. A pleural friction rub may be heard.)

The nurse is preparing to suction an adult client through the client's tracheostomy tube. Which interventions should the nurse perform for this procedure? *Select all that apply.* 1.Apply suction for up to 10 seconds. 2.Hyperoxygenate the client before suctioning. 3.Set the wall suction unit pressure at 160 mm Hg. 4.Apply suction while gently inserting the catheter. 5.Apply intermittent suction while rotating and withdrawing the catheter. 6.Advance the catheter until resistance is met and then pull the catheter back 1 cm.

1,2,5,6 (Rationale: Intermittent suction is applied while rotating the catheter for up to 10 seconds. The nurse should hyperoxygenate the client with a resuscitator bag/Ambu-bag connected to an oxygen source before suctioning because suction depletes the client's oxygen supply (option 2). The catheter should be inserted gently until resistance is met or the client coughs, then pulled back 1 cm or ½ inch. Intermittent suction is applied while rotating and withdrawing the catheter. Option 3 is incorrect because wall suction should be set to 80 mm Hg to 120 mm Hg. Pressure set at a higher level can cause trauma to respiratory tract tissues. Strict asepsis needs to be maintained, and the nurse would wear sterile gloves to perform this procedure. Suction is never applied when inserting the catheter because it will deplete oxygen and can traumatize tissues.)

The nurse is told that an assigned client will have a fenestrated tracheostomy tube inserted. The nurse plans care knowing that which facts are true with the use of a fenestrated tracheostomy tube? *Select all that apply.* 1.Enables the client to speak 2.Is necessary for mechanical ventilation 3.Must have the cuff deflated when capped 4.Eliminates the need for tracheostomy care 5.Prevents air from being inhaled through the tracheostomy opening

1,3 (Rationale: A fenestrated tracheostomy tube is used when a client is being weaned from breathing through the tracheostomy to breathing normally through the nose and mouth. A fenestrated tube has a small opening in the outer cannula that allows some air to escape through the larynx; this type of tube enables the client to speak. The cuff of the tracheostomy tube must always be deflated before the fenestrated tube is capped. When the cuff is inflated, the tracheostomy tube can be used for mechanical ventilation. When the cuff is deflated and the cap is applied, the client can breathe around the tracheostomy tube. The client continues to need cleaning of the tracheostomy site. The client is unable to breathe through the tracheal opening or at all if the cuff is inflated and the opening capped.)

The nurse is preparing a list of homecare instructions for the client who has been hospitalized and treated for tuberculosis. Which instructions should the nurse reinforce? *Select all that apply.* 1.Activities should be resumed gradually. 2.Avoid contact with other individuals except family members for at least 6 months. 3.A sputum culture is needed every 2 to 4 weeks once medication therapy is initiated. 4.Respiratory isolation is not necessary because family members have already been exposed. 5.Cover the mouth and nose when coughing or sneezing and confine used tissues to plastic bags. 6.When one sputum culture is negative, the client is no longer considered infectious and can usually return to his or her former employment.

1,3,4,5 (Rationale: The nurse should provide the client and family with information about tuberculosis and allay concerns about the contagious aspect of the infection. The client is reassured that after 2 to 3 weeks of medication therapy, it is unlikely that the client will infect anyone. The client is also informed that activities should be resumed gradually. The client and family are informed that respiratory isolation is not necessary because family members have already been exposed. The client is instructed about thorough hand washing, to cover the mouth and nose when coughing or sneezing, and to confine used tissues to plastic bags. The client is informed that a sputum culture is needed every 2 to 4 weeks once medication is initiated and that when the results of three sputum cultures are negative, the client is no longer considered infectious and can usually return to his or her former employment.)

Which are signs and symptoms characteristic of emphysema? *Select all that apply.* 1.Cyanosis 2.Wheezing 3.Weight loss 4.Barrel chest 5.Shortness of breath 6.Decreased lung sounds

1,3,4,5,6 (Rationale: The client with emphysema has a barrel chest, weight loss, and decreased lung sounds. Late signs and symptoms include shortness of breath and cyanosis. Wheezing is absent but is noted in other conditions such as asthma.)

The nurse is providing endotracheal suctioning to a client who is mechanically ventilated when the client becomes restless and tachycardic. Which actions should the nurse take? Select all that apply. 1.Notify the RN. 2.Notify the Rapid Response Team. 3.Finish the suctioning as quickly as possible. 4.Discontinue suctioning until the client is stabilized. 5.Contact the respiratory department to suction the client.

1,4 (Rationale: When suctioning a client with an endotracheal tube, the nurse removes the secretions and clears the airway. If a client becomes cyanotic or restless or develops tachycardia, bradycardia, or another abnormal heart rhythm, the nurse must discontinue suctioning until the client is stabilized. The nurse would also notify the RN. It is also important to monitor the vital signs and the pulse oximetry. If the client's condition continues to deteriorate, then the respiratory department and PHCP may need to be notified. There is no data in the question that indicates that the rapid response team needs to be notified.)

A client who has been taking isoniazid for 1½ months complains to the nurse about numbness, paresthesia, and tingling in the extremities. The nurse interprets that the client is experiencing which adverse effect? 1.Hypercalcemia 2.Peripheral neuritis 3.Small blood vessel spasm 4.Impaired peripheral circulation

2 (Rationale: An adverse effect of isoniazid is peripheral neuritis. This is manifested by numbness, tingling, and paresthesia in the extremities. This adverse effect can be minimized with pyridoxine (vitamin B6) intake.)

The nurse is caring for a restless client who keeps biting down on an orotracheal tube. The nurse uses which intervention to prevent the client from obstructing the airway with the teeth? 1.Bite stick 2.Oral airway 3.Nasal airway 4.Padded tongue blade

2 (Rationale: An oral airway may be used to keep the client from biting down, occluding an orotracheal tube. A nasal airway is not used in conjunction with an oral endotracheal tube. A padded tongue blade or a bite stick may be used initially to open the mouth for easier insertion of an oral airway.)

The nursing student and clinical instructor are performing tracheotomy suction at the bedside of an adult client with a tracheostomy. Which action by the nursing student is incorrect, causing the clinical instructor to intervene? 1.The student uses wall suction unit pressure of 100 mm Hg. 2.The student suctions the client's tracheotomy tube for 15 seconds. 3.The student places the client in semi-Fowler's position before suctioning. 4.The student inserts the catheter into the tracheostomy without applying suction.

2 (Rationale: Applying suction longer than 10 seconds can cause oxygen deprivation. The client should be placed into semi-Fowler's position to optimize breathing. Wall suction pressure of 100 mm Hg is usually recommended to prevent tissue disruption. The student is expected to insert the catheter without suction applied to maintain oxygen delivery and to prevent damage to the mucosa.)

The nurse is caring for a client with emphysema receiving oxygen. The nurse should consult with the registered nurse if the oxygen flow rate exceeded how many L/min of oxygen? 1.1 L/min 2.2 L/min 3.6 L/min 4.10 L/min

2 (Rationale: Between 1 L/min and 3 L/min of oxygen by nasal cannula may be required to raise the PaO2 level to 60 mm Hg to 80 mm Hg. However, oxygen is used cautiously in the client with emphysema and should not exceed 2 L/min unless specifically prescribed. Because of the long-standing hypercapnia that occurs in this disorder, the respiratory drive is triggered by low oxygen levels rather than by increased carbon dioxide levels, which is the case in a normal respiratory system.)

In which area of the chest should the nurse expect to auscultate this breath sound? (Refer to audio.) 1.Over the trachea 2.Over the peripheral lung fields 3.Posteriorly at T4 medial to the scapula 4.Between the first and second intercostal spaces at the sternal border anteriorly

2 (Rationale: Breath sounds are noises resulting from the transmission of vibrations produced by the movement of air in the respiratory passages. Normal breath sounds include bronchovesicular sounds, vesicular breath sounds, and bronchial breath sounds. The sounds that the nurse hears are vesicular breath sounds. Vesicular breath sounds normally are heard over the lesser bronchi, bronchioles, and lobes (peripheral lung fields). These sounds are soft and low pitched and resemble a sighing or gentle rustling, and the inspiration phase is longer than the expiration phase. Bronchovesicular breath sounds normally are heard over the first and second intercostal spaces at the sternal border anteriorly and at T4 medial to the scapula posteriorly (over major bronchi). These sounds are a mixture of bronchial and vesicular breath sounds and are moderately pitched with a medium intensity. The inspiration and expiration phases are equal. Bronchial breath sounds are loud, high-pitched sounds that resemble air blowing through a hollow pipe. The expiration phase is louder and longer than the inspiration phase, and there is a distinct pause between the inspiration and expiration phase. Bronchial breath sounds are heard normally over the manubrium.)

The nurse is assisting a client with a closed chest tube drainage system to get out of bed to a chair. During the transfer, the chest tube gets caught in the leg of the chair and accidentally dislodges from the insertion site. Which action should the nurse implement? 1.Change the dressing site on the chest. 2.Cover the insertion site with sterile gauze. 3.Reinsert the chest tube using sterile technique. 4.Transfer the client back to bed and encourage the client to breathe deeply.

2 (Rationale: If a chest tube dislodges from the insertion site, the nurse immediately covers the site with sterile Vaseline gauze. The nurse would then notify the registered nurse, assist the client back to bed, and perform a respiratory assessment on the client. The registered nurse would then contact the primary health care provider. The nurse does not reinsert a chest tube. The primary health care provider will reinsert the chest tube if necessary.)

A cardiac monitor alarm sounds, and the nurse notes a straight line on the monitor screen. What is the nurse's *immediate* nursing action? 1.Call a code. 2.Check the client. 3.Confirm the rhythm. 4.Check the cardiac leads and wires.

2 (Rationale: If a monitor alarms sounds, the nurse should first assess the clinical status of the client to see whether the problem is an actual dysrhythmia or a malfunction of the monitoring system. Asystole should not be mistaken for an unattached electrocardiogram wire. The other options would be appropriate once the nurse has checked the client.)

The low-pressure alarm sounds on the ventilator. The nurse checks the client and then attempts to determine the cause of the alarm but is unsuccessful. Which *initial* action should the nurse take? 1.Administer oxygen. 2.Ventilate the client manually. 3.Check the client's vital signs. 4.Start cardiopulmonary resuscitation (CPR).

2 (Rationale: If an alarm is sounding at any time and the nurse cannot quickly ascertain the problem, the client is disconnected from the ventilator and a manual resuscitation device is used to support respirations until the problem can be corrected. Although oxygen is helpful, it will not provide ventilation to the client. Checking vital signs is not the initial action. There is no reason to begin CPR.)

The nurse checks a closed chest tube drainage system on a client who had a lobectomy of the left lung 24 hours ago. The nurse notes that there has been no chest tube drainage for the past hour. Which should the nurse do *first*? 1.Contact the registered nurse. 2.Check for kinks in the chest drainage system. 3.Check the client's blood pressure and heart rate. 4.Connect a new drainage system to the client's chest tube.

2 (Rationale: If the nurse notes that a chest tube is not draining, the nurse would first check for a kink or possible clot in the chest drainage system. The nurse then notifies the registered nurse and observes the client for respiratory distress or mediastinal shift (if this occurs, the primary health care provider is notified). Checking the heart rate and blood pressure is not directly related to the lack of chest tube drainage. Connecting a new drainage system to the client's chest tube is done when the fluid drainage chamber is full. There is a specific procedure to follow when a new drainage system is connected to a client's chest tube.)

The low-exhaled volume (low-pressure) alarm sounds on a ventilator. The nurse rushes to the client's room and checks the client to determine the cause of the alarm but is unable to do so. Which would be the next immediate nursing action? 1.Call the rapid response team overhead. 2.Ventilate the client with a resuscitation bag. 3.Call the respiratory therapist to the bedside. 4.Call the client's primary health care provider to the bedside.

2 (Rationale: Mechanical ventilators have alarm systems that warn the nurse of a problem with either the client or the ventilator. Alarm systems must be activated and functional at all times. The nurse must recognize an emergency and intervene promptly so that complications are prevented. If the cause of an alarm cannot be determined, the nurse ventilates the client manually with a resuscitation bag until the problem is corrected by a second nurse, the respiratory therapist, or primary health care provider. The nurse also notifies the registered nurse (RN) of the occurrence and obtains assistance from the RN.)

The nurse is caring for a client with a tracheostomy tube who is receiving mechanical ventilation. The nurse is monitoring for complications related to the tracheostomy and suspects tracheoesophageal fistula when which occurs? 1.Suctioning is required frequently. 2.Aspiration of gastric contents occurs when suctioning. 3.The client's skin and mucous membranes are light pink. 4.Excessive secretions are suctioned from a tracheostomy.

2 (Rationale: Necrosis of the tracheal wall in a client with a tracheostomy can lead to an artificial opening between the posterior trachea and esophagus. This problem is called tracheoesophageal fistula. The fistula allows air to escape into the stomach causing abdominal distention. It also causes aspiration of gastric contents. Options 1, 3, and 4 are not signs of this complication.)

The nurse instructs a client on pursed-lip breathing and asks the client to demonstrate the breathing technique. Which observation by the nurse indicates that the client is performing the technique correctly? 1.The client breathes in through the mouth. 2.The client breathes out slowly through the mouth. 3.The client avoids using the abdominal muscles to breathe out. 4.The client puffs out the cheeks when breathing out through the mouth.

2 (Rationale: Pursed-lip breathing facilitates maximal expiration for clients with obstructive lung disease. The client should close the mouth and breathe in through the nose. The client then purses the lips and breathes out slowly through the mouth without puffing the cheeks. The client should spend at least twice the amount of time breathing out that it took to breathe in. The client should use the abdominal muscles to assist in squeezing out all of the air. The client also is instructed to use this technique during any physical activity, to inhale before beginning the activity, and to exhale while performing the activity. The client should never hold his or her breath.)

The nurse is caring for a client after pulmonary angiography via catheter insertion into the left groin. The nurse monitors for an allergic reaction to the contrast medium by observing for the presence of which? 1.Hypothermia 2.Respiratory distress 3.Hematoma in the left groin 4.Discomfort in the left groin

2 (Rationale: Signs of allergic reaction to the contrast medium include localized itching and edema, respiratory distress, stridor, and decreased blood pressure. Hypothermia is an unrelated event. Hematoma formation is a complication of the procedure, but does not indicate an allergic reaction. Discomfort is expected.)

The nurse is caring for a client hospitalized with acute exacerbation of chronic obstructive pulmonary disease (COPD). Which should the nurse expect to note? 1.Hypocapnia 2.Hyperinflated lungs on chest x-ray 3.Increased oxygen saturation with exercise 4.A widened diaphragm noted on chest x-ray

2 (Rationale: Signs/symptoms of chronic obstructive pulmonary disease include hypoxemia, hypercapnia, dyspnea on exertion and at rest, oxygen desaturation with exercise, and the use of accessory muscles of respiration. Chest x-ray will reveal a hyperinflated chest and a flattened diaphragm if the disease is advanced.)

The emergency department nurse is caring for a client who sustained a blunt injury to the chest wall. Which sign noted in the client indicates the presence of a pneumothorax? 1.Bradypnea 2.Shortness of breath 3.A low respiratory rate 4.The presence of a barrel chest

2 (Rationale: The client has sustained a blunt or a closed chest injury. This type of injury can result in a closed pneumothorax. Basic symptoms of a closed pneumothorax are shortness of breath and chest pain. A larger pneumothorax may present with tachypnea, cyanosis, diminished breath sounds, and subcutaneous emphysema. There may also be hyperresonance on the affected side. The presence of a barrel chest is characteristic of chronic obstructive pulmonary disease or emphysema.)

The nurse is talking with a client who is going to have a radical neck dissection and total laryngectomy. Which client statement indicates a need for further teaching concerning postoperative management? 1."I will gradually eat oral fluids and food." 2."I will require a lot of pain med after surgery." 3."I will need to support my head when I sit up." 4."I will determine options to restore some form of speech."

2 (Rationale: The client who is going to have a radical neck dissection and total laryngectomy needs to support the head when getting up to prevent stress on the suture line. Clients will have less pain than expected because many nerves have been cut, and there will be areas of permanent numbness. Although the client will initially be on tube feeding, the client will gradually be able to eat oral fluids and food. The client will later determine options to restore some form of speech.)

A client with a tracheostomy gets easily frustrated when trying to communicate personal needs to the nurse. The nurse determines that which method for communication may be the easiest for the client? 1.Use a pad and paper. 2.Use a picture or word board. 3.Have the family interpret needs. 4.Devise a system of hand signals.

2 (Rationale: The client with a tracheostomy in place cannot speak. The nurse devises an alternative communication system with the client. Use of a picture or word board is the simplest method of communication because it requires only pointing at the word or object. A pad and pencil is an acceptable alternative but requires more client effort and more time. The use of hand signals may not be a reliable method because it may not meet all needs and is subject to misinterpretation. The family does not need to bear the burden of communicating the client's needs, and they may not understand them either.)

Which statement by the client indicates a *need for further teaching* regarding the reinforced home care instructions for acute sinusitis? 1."I should drink large amounts of fluids." 2."I will need surgery to drain my sinuses." 3."I should apply a wet, warm heat pack over my sinuses." 4."I will need to sleep with the head of the bed elevated."

2 (Rationale: The nurse provides instructions to the client regarding measures to promote sinus drainage, comfort, and resolution of the infection. The nurse instructs the client to apply heat in the form of hot wet packs over the affected sinuses to promote comfort and help resolve the infection. Large amounts of fluids are important to help liquefy secretions. Sleeping with the head of the bed elevated to a 45-degree angle will assist in promoting drainage. Surgery may be performed to improve drainage in chronic conditions if other measures are not helpful, but it is not usually a treatment measure for acute sinusitis.)

The nurse is monitoring a client with a closed chest tube drainage system and notes fluctuation of the fluid level in the water-seal chamber during inspiration and expiration. On the basis of this finding, which conclusion should the nurse make? 1.There is a leak in the system. 2.The chest tube is functioning as expected. 3.The amount of suction needs to be decreased. 4.The occlusive dressing at the insertion site needs reinforcement.

2 (Rationale: The presence of fluctuation of the fluid level in the water-seal chamber indicates a patent drainage system. With normal breathing, the water level rises with inspiration and falls with expiration. Fluctuation stops if the tube is obstructed, if the suction is not working properly, or if the lung has reexpanded. The remaining options are incorrect interpretations of the finding.)

The nurse is performing nasopharyngeal suctioning on a client and suddenly notes the presence of bloody secretions. Which action should the nurse implement? 1.Continue suctioning to remove the blood. 2.Check the amount of suction pressure being applied. 3.Encourage the client to cough out the bloody secretions. 4.Remove the suction catheter from the nose and begin vigorous suctioning through the mouth.

2 (Rationale: The return of bloody secretions is an unexpected outcome related to suctioning. If this occurs, the nurse should first assess the client and then determine the amount of suction pressure being applied. The amount of suction pressure may need to be decreased. The nurse also needs to be sure that intermittent suction and catheter rotation are being done during suctioning. Continuing with the suctioning or vigorous suctioning through the mouth will cause increased trauma and thus increased bleeding. Suctioning is normally performed on clients who are unable to expectorate secretions. Therefore, it is unlikely that the client will be able to cough out the bloody secretions.)

The nurse is listening to the client's breath sounds and hears musical whistling noises on inspiration and expiration scattered throughout the right lung fields. How should the nurse interpret these noises? 1.Crackles 2.Wheezes 3.Rhonchi 4.Pleural friction rub

2 (Rationale: Wheezes are musical noises heard on inspiration, expiration, or both. They are the result of narrowed air passages. Crackles have the sound that is heard when a few strands of hair are rubbed together near the ear and indicate fluid in the alveoli. Rhonchi are usually heard on expiration when there is excessive production of mucus that accumulates in the air passages. A pleural friction rub is characterized by sounds that are described as creaking, groaning, or grating in quality. The sounds are localized over an area of inflammation of the pleura and may be heard in both the inspiratory and expiratory phases of the respiratory cycle.)

The nurse is assisting in planning care for a client with a chest tube. The nurse should suggest to include which interventions in the plan? *Select all that apply.* 1.Pin the tubing to the bed linens. 2.Be sure all connections remain airtight. 3.Be sure all connections are taped and secure. 4.Monitor closely for tubing that is kinked or obstructed. 5.Empty the drainage from the drainage collection chamber daily.

2, 3, 4 (Rationale: The chest tube system must be maintained as a closed system in order for the air to be removed by suction and for the lungs to reexpand to a normal state. The connections should be air tight (no leaks), and all connections should be tapes and secure. It is important that the tubes to the suction and the chest tube be patent (without kinks or obstructions). Chest-tube tubing is never pinned to the bed linens because this presents the risk of accidental dislodgment of the tube when the client moves. The chest tube system is not opened and emptied because a closed system must be maintained; if the system is opened, air pressure causes air to rush in, and lung collapse can occur.)

The nurse is assigned to assist the primary health care provider (PHCP) with the removal of a chest tube. Which interventions should the nurse anticipate performing during this process? *Select all that apply.* 1.Reinforce instructions to breathe deeply while the tube is removed. 2.Cover the site with an occlusive dressing after the tube is removed. 3.Clamp the chest tube near the insertion site just before the removal. 4.Raise the drainage system to the level of the chest tube insertion site. 5.Have the client perform the Valsalva maneuver as the chest tube is pulled out.

2, 5 (Rationale: A chest tube is removed when the lung has fully reexpanded or there is limited drainage. When the chest tube is removed, the client is asked to perform a Valsalva maneuver (i.e., take a deep breath, exhale, and bear down), the tube is quickly withdrawn, and an airtight (occlusive) dressing is taped in place. An alternative instruction is to ask the client to take a deep breath and hold the breath while the tube is removed. After the tube is removed, the client should take deep breaths to ensure adequate lung expansion. The tube is not usually clamped before it is removed, and the drainage apparatus must always be lower than the chest tube site.)

The nurse is caring for a client hospitalized with acute exacerbation of chronic obstructive pulmonary disease (COPD). Which should the nurse expect to note in this client? *Select all that apply.* 1.Hypocapnia 2.Dyspnea during exertion 3.Presence of a productive cough 4.Difficulty breathing while talking 5.Increased oxygen saturation with exercise 6.A shortened expiratory phase of respiration

2,3,4 (Rationale: Clinical manifestations of COPD include hypoxemia, hypercapnia, and dyspnea during exertion and at rest, oxygen desaturation with exercise, use of accessory muscles of respiration, and a prolonged expiratory phase of respiration. The client may also exhibit difficulty breathing while talking, and may have to take breaths between every one or two words. Some clients with COPD, especially those with a history of smoking, often have a productive cough particularly when arising in the morning. The chest x-ray will reveal a hyperinflated chest and a flattened diaphragm if the disease is advanced.)

A client with tuberculosis (TB) asks the nurse about precautions to take after discharge from the hospital to prevent transmitting infection to others. Which statements indicate prevention of transmission of tuberculosis? *Select all that apply.* 1."I will bleach my clothes and bedding after use." 2."My family and I will practice good hand hygiene." 3."I will discard disposable tissues into a plastic bag." 4."I will cover my mouth when I cough, sneeze, or laugh." 5."All the deep pile carpeting will be removed from my home."

2,3,4 (Rationale: TB is spread by droplet nuclei or by the airborne route. The disease is not carried on objects such as clothing, eating utensils, linens, or furniture. Bleaching of clothing and linens is unnecessary, although the client and family members should use good hand-washing technique. It is unnecessary to remove carpeting from the home. The client should protect others by covering the mouth when coughing, laughing, or sneezing. The client and family should wash hands often. Disposable tissues should be used and discarded in a plastic bag.)

The nurse is assigned to assist in caring for a client with a chest tube drainage system. Which interventions should the nurse implement? *Select all that apply.* 1.Pin excess tubing to the bedclothes. 2.Check for subcutaneous emphysema. 3.Empty the chest tube drainage at the end of the shift. 4.Check to see that the chest tube drainage is fluctuating. 5.Maintain chest tube drainage container below the client's chest.

2,4,5 (Rationale: Remember that the pleural cavity is an airtight compartment. The apparatus and all connections must remain airtight at all times. Do not allow the tubing to become kinked or obstructed by the weight of the client. Never pin the tubing to the bedclothes. Do not empty chest tube drainage containers. The system must remain closed. The system operates by gravity and must remain below the client's chest level at all times. Fluid in the chamber should fluctuate with inhalation and exhalation. A "puffed-up" appearance of the client's chest or neck could be subcutaneous emphysema.)

The nurse is collecting data from a client who is experiencing the typical signs/symptoms of tuberculosis (TB). Which are signs and symptoms of tuberculosis? *Select all that apply.* 1.Weight gain 2.Night sweats 3.Sporadic coughing 4.Mucopurulent sputum 5.Afternoon low grade fever

2,4,5 (Rationale: The client with tuberculosis may report symptoms that have been present for weeks or even months. The symptoms may include fatigue, lethargy, chest pain, anorexia and weight loss, night sweats, low-grade fever in the afternoon, and cough with mucoid or blood-streaked sputum. The cough is often persistent.)

A client with pneumonia is admitted to the hospital, and the primary health care provider writes prescriptions for the client. Which prescription should the nurse complete *first*? 1.Increase the intake of oral fluids. 2.Administer a prescribed antibiotic. 3.Obtain a culture and sensitivity of sputum. 4.Encourage the use of an incentive spirometer.

3 (Rationale: A culture and sensitivity should be obtained before any antibiotic therapy is begun to avoid masking the microorganisms identified in the culture. Options 1, 2, and 4 are standard parts of therapy for pneumonia, but sputum is collected first.)

The nurse is gathering data on a client with a diagnosis of tuberculosis. The nurse should review the results of which diagnostic test to confirm this diagnosis? 1.Chest x-ray 2.Bronchoscopy 3.Sputum culture 4.Tuberculin skin test

3 (Rationale: A definitive diagnosis of tuberculosis is confirmed through culture and isolation of Mycobacterium tuberculosis. A presumptive diagnosis is made on the basis of a tuberculin skin test, a sputum smear that is positive for acid-fast bacteria, a chest x-ray, and histological evidence of granulomatous disease on biopsy.)

A client has just returned from intrathoracic surgery where a chest tube was placed. The nurse notes a small amount of serosanguineous drainage on the chest tube's dressing. Which action should the nurse take? 1.Notify the surgeon. 2.Change the dressing. 3.Reinforce the dressing. 4.Document the findings.

3 (Rationale: Chest tube dressings may be reinforced but are not changed unless prescribed by the surgeon. Dressings are kept dry and occlusive. A small amount of serosanguineous drainage right after surgery can be expected, so the surgeon would not be notified. Just documenting the findings does not ensure a dry dressing.)

The nurse is performing nasotracheal suctioning of a client. The nurse interprets that the client is adequately tolerating the procedure if which observation is made? 1.Skin color becomes cyanotic. 2.Secretions are becoming bloody. 3.Coughing occurs with suctioning. 4.Heart rate decreases from 78 beats/minute to 54 beats/minute.

3 (Rationale: Coughing is a normal response to suctioning for the client with an intact cough reflex, and it is not an indication that the client is not tolerating the procedure. The client should be encouraged to cough to help with removal of secretions from the lungs. The nurse should monitor for the adverse effects of suctioning, which include cyanosis (pulse oximetry falls below 90% or 5% from baseline), excessively rapid or slow heart rate (a 20 beat/minute change), or the sudden development of bloody secretions. If they occur, the nurse stops suctioning, administers oxygen as appropriate, and reports these signs to the PHCP immediately.)

The nurse is caring for an older client who is on bed rest. The nurse plans which intervention to prevent respiratory complications? 1.Decreasing oral fluid intake 2.Monitoring the vital signs every shift 3.Changing the client's position every 2 hours 4.Instructing the client to bear down every hour and to hold his or her breath

3 (Rationale: Frequent position changes help mobilize lung secretions and prevent pooling. This is the only intervention identified in the options that will prevent respiratory complications. The nurse should encourage fluid intake to thin secretions and thus enable the client to expectorate more easily. It is important to encourage coughing and deep breathing to mobilize lung secretions. The nurse should assess the client's vital signs every 4 hours to identify an elevated temperature, which may suggest infection. The client should be instructed to avoid the Valsalva maneuver or any activity that involves holding the breath.)

The nurse is providing morning care to a client who has a closed chest tube drainage system to treat a pneumothorax. When the nurse turns the client to the side, the chest tube is accidentally dislodged from the chest. After immediately applying sterile gauze over the chest tube insertion site which should the nurse do *next*? 1.Replace the chest tube system. 2.Obtain a pulse oximetry reading. 3.Notify the registered nurse (RN). 4.Place the client in Trendelenburg's position.

3 (Rationale: If the chest drainage system is dislodged from the insertion site, the nurse immediately applies sterile gauze over the site and notifies the RN, who then calls the primary health care provider (PHCP). The nurse should maintain the client in an upright position. A new chest tube system may be attached if the tube requires insertion, but this would not be the next action. Pulse oximetry readings should assist in determining the client's respiratory status, but the priority action should be to notify the RN, who will then call the PHCP)

The nurse is caring for several clients with respiratory disorders. Which client is at least risk for developing a tuberculosis infection? 1.An uninsured man who is homeless 2.A woman newly immigrated from Korea 3.A man who is an inspector for the U.S. Postal Service 4.An older woman admitted from a long-term care facility

3 (Rationale: People at high risk for acquiring tuberculosis include children younger than 5 years of age; homeless individuals or those from a lower socioeconomic group, minority groups, or immigrant group; individuals in constant, frequent contact with an untreated or undiagnosed individual; individuals living in crowded areas such as long-term care facilities, prisons, and mental health facilities; older clients; malnourished individuals, those with an infection, or an immune dysfunction or human immunodeficiency virus infection, or individuals who are immunosuppressed as a result of medication therapy; and individuals who abuse alcohol or are IV drug users.)

A client with no history of respiratory disease is admitted to the hospital with respiratory failure. The nurse reviews the arterial blood gas reports for which results that are consistent with this disorder? 1.Pao2 58 mm Hg, Paco2 32 mm Hg 2.Pao2 60 mm Hg, Paco2 45 mm Hg 3.Pao2 49 mm Hg, Paco2 52 mm Hg 4.Pao2 73 mm Hg, Paco2 62 mm Hg

3 (Rationale: Respiratory failure is described as a Pao2 of 50 mm Hg or less, and a Paco2 of 50 mm Hg or greater in a client with no history of respiratory disease. In a client with a history of respiratory disorder with hypercapnia, Paco2 elevations of 5 mm Hg or more from the client's baseline are considered diagnostic.)

A client has a prescription for continuous monitoring of oxygen saturation by pulse oximetry. The nurse performs which *best* action to ensure accurate readings on the oximeter? 1.Apply the sensor to a finger that is cool to the touch. 2.Apply the sensor to a finger with very dark nail polish. 3.Ask the client to limit motion in the hand attached to the pulse oximeter. 4.Place the sensor distal to an intravenous (IV) site with a continuous IV infusion.

3 (Rationale: Several factors can interfere with the reading of accurate oxygen saturation levels on a pulse oximeter. To ensure accurate readings, the nurse should ask the client to limit motion of the area attached to the sensor. The nurse should apply the device to a warm area because hypotension, hypothermia, and vasoconstriction interfere with blood flow to the area. If possible, the nurse should avoid placing the sensor distal to any invasive arterial or venous catheters, pressure dressings, or blood pressure cuffs. The nurse needs to know that very dark nail polish (black, brown-red, blue, green) interferes with accurate measurement.)

A client arrives in the emergency department with an episode of status asthmaticus. What is the nurse's *priority* action? 1.Obtain a set of vital signs. 2.Administer oxygen at 21%. 3.Place the client in high-Fowler's position. 4.Obtain equipment for starting an intravenous line.

3 (Rationale: The first nursing action is to place the client in a position that aids in respiration, which would be sitting bolt upright or in high-Fowler's position. Other nursing actions follow in rapid sequence and include monitoring vital signs and administering bronchodilators and oxygen (but at levels of 2 to 5 L/min or 24% to 28% by Ventimask). Insertion of an intravenous line and ongoing monitoring of respiratory status are also indicated.)

A client arrives in the emergency department with a bloody nose. Which is the initial nursing action? 1.Place the client in a supine position. 2.Apply an ice collar around the client's neck. 3.Assist the client to a sitting position with the head tilted slightly forward. 4.Instruct the client to swallow the blood until the bleeding can be controlled.

3 (Rationale: The initial nursing action to treat the client with a bloody nose is to loosen clothing around the neck to prevent pressure on the carotid artery. The client should be assisted to a sitting position with the head tilted slightly forward, and pressure should be applied to the nares by pinching the nose toward the septum for 10 minutes. Ice packs can be applied to the nose and forehead. If these actions are not successful in controlling the bleeding, an ice collar may be applied along with a topical vasoconstrictive medication. The primary health care provider may also prescribe packing to the nostrils. The client should be provided with an emesis basin and should be instructed not to swallow blood to reduce the risk of nausea and vomiting.)

A client is at risk of developing a pulmonary embolism. The nurse monitors for which *initial* sign/symptom? 1.Hot, flushed feeling 2.Sudden chills and fever 3.Chest pain that occurs suddenly 4.Dyspnea noted when deep breaths are taken

3 (Rationale: The most common initial symptom in pulmonary embolism is chest pain that is sudden in onset. The next most commonly reported symptom is dyspnea, which is accompanied by an increased respiratory rate. Other typical symptoms of pulmonary embolism include cough, tachycardia, fever, diaphoresis, anxiety, and possibly syncope)

The nurse is suctioning an adult client through a tracheostomy tube. During the procedure, the nurse notes that the client's oxygen saturation by pulse oximetry is 89%. Which action should the nurse implement? 1.Continue suctioning. 2.Call respiratory therapy. 3.Stop the suctioning procedure. 4.Obtain a smaller suction catheter.

3 (Rationale: The nurse should monitor the client's heart rate and pulse oximetry during suctioning to assess the client's tolerance of the procedure. Oxygen desaturation below 90% indicates hypoxia. If hypoxia occurs during suctioning, the nurse stops the suctioning procedure. Using the 100% oxygen delivery system, the client is reoxygenated until baseline parameters are achieved. The size of the catheter should not exceed half of the size of the tracheal lumen. In adults, the standard catheter size is 12 to 14 French. Adequate catheter size facilitates efficient removal of secretions without causing hypoxemia.)

A client who has just suffered a flail chest is experiencing severe pain and dyspnea. Which would be the appropriate nursing action? 1.Reposition the client. 2.Document the findings. 3.Notify the registered nurse. 4.Medicate the client for pain.

3 (Rationale: The nurse would notify the registered nurse who would then contact the primary health care provider. The client with severe flail chest will have significant paradoxical chest movement. This causes the mediastinal structures to swing back and forth with respiration. This will lead to severe pain and dyspnea and can affect circulatory hemodynamics.)

The nurse is assessing a client who has frequent episodes of asthma. Which assessment finding is *most* closely associated with asthma? 1.Fine rhonchi 2.Pink, frothy sputum 3.Bilateral wheezing 4.Rhonchi that clear with a cough

3 (Rationale: Wheezing is the symptom most associated with asthma, a reactive airway disease. Fine rhonchi; rhonchi that clear with a cough; and pink, frothy sputum are not associated with asthma.)

The nurse is assisting with monitoring the functioning of a chest-tube drainage system in a client who just returned from the recovery room after a thoracotomy with wedge resection. Which findings should the nurse expect to note? *Select all that apply.* 1.Excessive bubbling in the water-seal chamber 2.Vigorous bubbling in the suction-control chamber 3.50 mL of drainage in the drainage-collection chamber 4.The drainage system is maintained below the client's chest. 5.An occlusive dressing is in place over the chest-tube insertion site. 6.Fluctuation of water in the tube of the water-seal chamber during inhalation and exhalation

3,4,5,6 (Rationale: In a thoracotomy the lung is opened and exposed, and a wedge resection is the removal of part of the lung. The chest tube is placed during the surgery to remove fluid and air so the remaining lung can reinflate. The bubbling of water in the water-seal chamber should be gentle and indicates air drainage from the client. This is usually seen when intrathoracic pressure is greater than atmospheric pressure, and it may occur during exhalation, coughing, or sneezing. The fluctuation of water in the tube in the water-seal chamber during inhalation and exhalation is expected. An absence of fluctuation may indicate that the chest tube is obstructed, the lung has reexpanded, or no more air is leaking into the pleural space. Gentle (not vigorous) bubbling should be noted in the suction-control chamber. A total of 50 mL of drainage is not excessive in a client returning to the nursing unit from the recovery room; however, drainage of more than 70 mL/hour to 100 mL/hour is considered excessive and requires RN and PHCP notification. The chest-tube insertion site is covered with an occlusive (airtight) dressing to prevent air from entering the pleural space. Positioning the drainage system below the client's chest allows gravity to drain the pleural space. Excessive and/or vigorous bubbling in the water-seal chamber may indicate an air leak, which is an unexpected finding.)

The nurse is assigned to care for a client who has a chest tube. The nurse is told to monitor the client for crepitus (subcutaneous emphysema). Which method should be used to monitor the client for crepitus? 1.Auscultating the posterior breath sounds 2.Asking the client about pain upon inspiration 3.Placing the hands over the rib area and observing expansion 4.Palpating the skin around the chest and neck for a crackling sensation

4 (Rationale: Air caught under the skin in the subcutaneous tissues is known as crepitus or subcutaneous emphysema. It presents as a "puffed-up" appearance that is caused by the leakage of air into the subcutaneous tissues. It is monitored by palpating, and it feels like bubble wrap when palpated. Auscultation of posterior breath sounds gives data about adequate depth of respirations. Pain upon inspiration can occur with pleurisy (inflammation of the pleurae) or pericarditis. Placing the hands over the rib area is a method of determining equal chest expansion on each side.)

A client is admitted to the hospital with a diagnosis of carbon dioxide narcosis. In addition to respiratory failure, the nurse plans to monitor the client for which complication of this disorder? 1.Paralytic ileus 2.Hypernatremia 3.Hyperglycemia 4.Increased intracranial pressure

4 (Rationale: Carbon dioxide acts as a vasodilator to cerebral blood vessels. With a sufficient rise in carbon dioxide, the client may suffer increased intracranial pressure, which is reflected initially as papilledema and dilated conjunctival blood vessels. Options 1, 2, and 3 are not complications.)

A client with pneumonia is experiencing problems with ventilation as a result of accumulated respiratory secretions. The nurse determines that which data accurately indicate *effectiveness* of the treatments prescribed for this problem? 1.Venous oxygen saturation is 95%. 2.Respiratory rate is 20 breaths per minute. 3.Client demonstrated effective coughing techniques. 4.Arterial blood gases indicate a pH of 7.4, Po2 of 80 mm Hg, and Pco2 of 40 mm Hg.

4 (Rationale: Demonstration of adequate ventilation can only be accurately evaluated when both Po2 and Pco2 levels are known. The other options do not indicate adequate gas exchange. Remember that oxygen saturation index is a measure of the percent of oxygen attached to the available hemoglobin.)

A client is admitted to the hospital with acute exacerbation of chronic obstructive pulmonary disease (COPD). Which arterial blood gas supports this diagnosis? 1.Po2 of 68 mm Hg and Pco2 of 40 mm Hg 2.Po2 of 55 mm Hg and Pco2 of 40 mm Hg 3.Po2 of 70 mm Hg and Pco2 of 50 mm Hg 4.Po2 of 60 mm Hg and Pco2 of 50 mm Hg

4 (Rationale: During an acute exacerbation, the arterial blood gases deteriorate with decreasing Po2 levels and increasing Pco2 levels. In the early stages of chronic obstructive pulmonary disease, arterial blood gases demonstrate mild to moderate hypoxemia with the Po2 in the high 60s to high 70s (mm Hg) and normal arterial Pco2. As the condition advances, hypoxemia increases and hypercapnia may result.)

A client being discharged from the hospital to home with a diagnosis of tuberculosis is worried about the possibility of infecting family members and others. Which information should reassure the client that contaminating family members and others is not likely? 1.The family does not need therapy, and the client will not be contagious after 1 month of medication therapy. 2.The family does not need therapy, and the client will not be contagious after 6 consecutive weeks of medication therapy. 3.The family will receive prophylactic therapy, and the client will not be contagious after 1 continuous week of medication therapy. 4.The family will receive prophylactic therapy, and the client will not be contagious after 2 to 3 consecutive weeks of medication therapy.

4 (Rationale: Family members or others who have been in close contact with a client diagnosed with tuberculosis are placed on prophylactic therapy with isoniazid for 6 to 12 months. The client is usually not contagious after taking medication for 2 to 3 consecutive weeks. However, the client must take the full course of therapy (for 6 months or longer) to prevent reinfection or drug-resistant tuberculosis.)

The nurse provides instructions to a client about the use of an incentive spirometer. The nurse determines that the client needs further teaching about its use if the client makes which statement? 1."I need to sit upright when using the device." 2."I will inhale slowly, maintaining a constant flow." 3."I need to place my lips completely over the mouthpiece." 4."After maximal inspiration, I will hold my breath for 10 seconds and then exhale."

4 (Rationale: For optimal lung expansion with the incentive spirometer, the client should assume a semi-Fowler's or high-Fowler's position. The mouthpiece should be covered completely and tightly while the client inhales slowly, with a constant flow through the unit. When maximal inspiration is reached, the client should hold the breath for 5 seconds and then exhale slowly through pursed lips.)

A client has a chest tube that is attached to a chest drainage system. The chest tube becomes disconnected. What should the nurse do *immediately*? 1.Assess lung sounds. 2.Clamp tube above open end. 3.Put gloved thumb over open end. 4.Put open end under sterile water.

4 (Rationale: If the chest tube becomes unattached, do not clamp the tube; place the end of the tubing in a container of sterile water. This creates a "water seal" and can prevent tension pneumothorax.)

A client attached to mechanical ventilation suddenly becomes restless and pulls out the tracheostomy tube. Which is the nurse's priority intervention? 1.Prepare for reintubation. 2.Call the rapid response team. 3.Call the registered nurse. 4.Check the client for spontaneous breathing.

4 (Rationale: If unexpected intubation occurs, the nurse would first check the client for airway patency, spontaneous breathing, and vital signs. The nurse would remain with the client, call for assistance from the registered nurse, and prepare for reintubation. There are no data in the question to indicate that a code needs to be called.)

The nurse is caring for a client who is being treated for a pneumothorax with a closed chest tube drainage system. When repositioning the client, the chest tube disconnects. Which nursing action should be immediate? 1.Clamp the chest tube. 2.Instruct the client to inhale. 3.Call the primary health care provider. 4.Reattach the chest tube to the drainage system.

4 (Rationale: In most situations, clamping chest tubes is contraindicated, and agency policy and procedure must be followed with regard to clamping a chest tube. When the client has a residual air leak or pneumothorax, clamping the chest tube may precipitate a tension pneumothorax because the air has no escape route. If the tube becomes disconnected, it is best to immediately reattach it to the drainage system or to submerge the end in a bottle of sterile water or saline to reestablish a water seal. If sterile water or saline is not readily available, it is preferable to leave the tube open because the risk of tension pneumothorax outweighs the consequences of an open tube. The nurse would also notify the registered nurse of the occurrence. The primary health care provider will need to be notified, but this is not the immediate action. The client would not be instructed to inhale.)

A postoperative client is using an incentive spirometer. The nurse observes the client inhale slowly with the mouthpiece placed between the teeth with the lips closed. The client inhales to the preset inspiratory goal and holds the breath for about 3 seconds, then exhales slowly. The client takes one breath and returns the incentive spirometer to the bedside. Based on this observation, which interpretation should the nurse make? 1.The client should be inhaling and exhaling quickly. 2.The client is using the incentive spirometer correctly. 3.The client should not be holding the breath following inhalation. 4.The client should be repeating the sequence 10 to 20 times in each session.

4 (Rationale: Incentive spirometer devices use a concept of sustained maximal inspiration. Each device has a means of setting an inspiratory goal. Correct use requires a spontaneous, slow, voluntary, deep breath. When full inhalation is reached, the breath is held for at least 3 seconds. This sequence is repeated 10 to 20 times an hour. Incentive spirometer exercises are most effective when used every hour while the client is awake.)

The nurse is reinforcing instructions to a hospitalized client with a diagnosis of emphysema about positions that will enhance the effectiveness of breathing during dyspneic episodes. Which position should the nurse instruct the client to assume? 1.Side-lying in bed 2.Sitting in a recliner chair 3.Sitting up in bed at a 90 degree angle 4.Sitting on the side of the bed leaning on an overbed table

4 (Rationale: Positions that will assist the client with breathing include sitting up and leaning on an overbed table, sitting up and resting with the elbows on the knees, or standing or leaning against the wall. The positions in options 1, 2, and 3 will not enhance the effectiveness of breathing.)

The nurse is instructing a client about pursed lip breathing, and the client asks the nurse about its purpose. The nurse should tell the client that the *primary* purpose of pursed lip breathing is which? 1.Promote oxygen intake 2.Strengthen the diaphragm 3.Strengthen the intercostal muscles 4.Promote carbon dioxide elimination

4 (Rationale: Pursed lip breathing facilitates maximal expiration for clients with obstructive lung disease and promotes carbon dioxide elimination. This type of breathing allows better expiration by increasing airway pressure, which keeps air passages open during exhalation. Options 1, 2, and 3 are not the purposes of this type of breathing.)

The nurse is assisting in caring for a client with pneumonia who suddenly becomes restless. Arterial blood gases are drawn, and the results reveal a Pao2 of 60 mm Hg. The nurse reviews the plan of care for the client and determines that which *priority* problem potentially exists for this client? 1.Fatigue 2.Aspiration 3.Airway obstruction 4.Ineffective gas exchange

4 (Rationale: Restlessness and low Pao2 are hallmark signs of ineffective oxygen exchange. Airway obstruction and aspiration are not problems that are specifically associated with existing pneumonia. Although many clients with pneumonia experience fatigue, this is not the priority problem.)

The nurse is reinforcing discharge instructions to the client with pulmonary sarcoidosis. The nurse knows that the client understands the information if the client verbalizes which *early* sign of exacerbation? 1.Fever 2.Fatigue 3.Weight loss 4.Shortness of breath

4 (Rationale: Shortness of breath is an early sign of exacerbation of pulmonary sarcoidosis. Others include chest pain, hemoptysis, and pneumothorax. Systemic signs and symptoms that occur later include weakness and fatigue, malaise, fever, and weight loss.)

The nurse is reinforcing discharge teaching to a client diagnosed with tuberculosis who has been taking medication for 1½ weeks. The nurse knows that the client has understood the information if which statement is made? 1."I can't shop at the mall for the next 6 months." 2."I need to continue medication therapy for 2 months." 3."I can return to work if a sputum culture comes back negative." 4."I should not be contagious after 2 to 3 weeks of medication therapy."

4 (Rationale: The client continues medication therapy for 6 to 12 months depending on the situation. The client is generally considered to not be contagious after 2 to 3 weeks of medication. The client is instructed to wear a mask if there will be exposure to crowds until the medication is effective in preventing transmission. The client is allowed to return to employment when the results of three sputum cultures are negative.)

The nurse is assisting in admitting a client who is suspected of having tuberculosis (TB) to the nursing unit. The nurse plans to admit the client to which type of room? 1.Venting through single filters and ultraviolet light 2.Natural lighting with three air exchanges per hour 3.One air exchange per hour and venting to the outside 4.Venting to the outside, six air exchanges per hour, and ultraviolet light

4 (Rationale: The client is admitted to a private room that has at least six air exchanges per hour and negative pressure in relation to surrounding areas. The room should be vented to the outside and should have ultraviolet lights installed.)

The nurse is assisting in collecting subjective and objective data from a client admitted to the hospital with tuberculosis (TB). The nurse should expect to note which finding? 1.High fever 2.Flushed skin 3.Complaints of weight gain 4.Complaints of night sweats

4 (Rationale: The client with tuberculosis usually experiences a low-grade fever, weight loss, pallor, chills, and night sweats. The client also will complain of anorexia and fatigue. Pulmonary symptoms include a cough that is productive of a scant amount of mucoid sputum. Purulent, blood-stained sputum is present if cavitation occurs. Dyspnea and chest pain occur late in the disease.)

The nurse is checking the chest tube drainage system of a postoperative client who had a right upper lobectomy. The closed drainage system has 300 mL of bloody drainage, and the nurse notes intermittent bubbling in the water-seal chamber. One hour following the initial data collection, the nurse notes that the bubbling in the water-seal chamber is now constant, and the client appears dyspneic. Based on these findings, which action should the nurse do *first*? 1.Check the client's vital signs. 2.Note the amount of drainage. 3.Check the client's lung sounds. 4.Inspect chest tube connections.

4 (Rationale: The client's dyspnea is most likely related to an air leak caused by a loose connection. Other causes might be a tear in the pulmonary pleura, which requires primary health care provider intervention. Although the other options are correct, they should be performed after initial attempts to locate and correct the air leak. It only takes a moment to check the connections, and if a leak is found and corrected, the client's symptoms should resolve. The nurse would also notify the registered nurse.)

The nurse is suctioning a client through a tracheostomy tube. During the procedure, the client begins to cough, and the nurse notes the presence of an audible wheeze. The nurse attempts to remove the suction catheter from the client's trachea but is unable to do so. What is the nurse's *priority* response? 1.Call a code. 2.Administer a bronchodilator. 3.Contact the registered nurse 4.Disconnect the suction source from the catheter.

4 (Rationale: The inability to remove a suction catheter is a critical situation. This finding, along with the client's symptoms presented in the question, indicates the presence of bronchospasm and bronchoconstriction. The nurse would immediately disconnect the suction source from the catheter but leave the catheter in the trachea. The nurse would then connect the oxygen source to the catheter. The nurse also notifies the registered nurse who then notifies the primary health care provider. The primary health care provider will most likely prescribe an inhaled bronchodilator. The nurse also prepares for emergency resuscitation if this situation occurs during suctioning.)

The nurse is reviewing the arterial blood gas results of an assigned client. Which arterial blood gases indicate metabolic alkalosis? 1.pH of 7.35, Pco2 of 50 mm Hg, HCO3- of 32 mEq/L 2.pH of 7.45, Pco2 of 35 mm Hg, HCO3- of 22 mEq/L 3.pH of 7.38, Pco2 of 45 mm Hg, HCO3- of 32 mEq/L 4.pH of 7.48, Pco2 of 40 mm Hg, HCO3- of 36 mEq/L

4 Rationale: In metabolic alkalosis, the pH is elevated along with the bicarbonate level. Option 4 is the only option that reflects these values.

The nurse is collecting respiratory data from an adult client and is auscultating for normal breath sounds. The nurse should expect to hear bronchial breath sounds in which anatomical area? Refer to figure. A. tracheal b. Bronchovesicular c. Vesicular

A (Bronchial sounds = tracheal sounds) (Rationale: There are three types of normal breath sounds in the adult and older child. These include bronchial (sometimes called "tracheal" or "tubular"), bronchovesicular, and vesicular sounds. Bronchial breath sounds are heard over the trachea and larynx. Bronchovesicular breath sounds are heard over the major bronchi. Vesicular breath sounds are heard over peripheral lung fields where the air enters the alveoli.)

The nurse should plan to fill which chamber of the chest drainage unit to prevent atmospheric air from reentering the pleural space? a) suction control chamber b) water seal chamber c) Collection chamber

b (Rationale: To prevent atmospheric air from reentering the client's pleural space, the nurse needs to fill the water seal chamber to the level prescribed by the manufacturer, usually 2 cm. This is the minimum amount of fluid needed to prevent atmospheric air from reentering the pleural space. Therefore, options 1, 3, and 4 are incorrect. Option 1 identifies the suction control chamber. Options 3 and 4 identify the collection chamber.)


Set pelajaran terkait

Chapter 18: Fetal Assessment during Labor

View Set

LEADING AND MANAGING, Taylor Ch. 14: Assessing, Taylor Ch. 16: Outcome Identification and Planning, Taylor Ch. 18: Evaluating, Taylor Ch. 8: Communication, Taylor Ch. 19: Documenting and Reporting, Taylor Ch. 17: Implementing, Taylor Ch. 13: Blended...

View Set

Principal management chapter 6-10

View Set

You're screwed part 2 (Mental health)

View Set

Physiological Psychology Test 31) Which of the following statements describes range fractionation?

View Set

Convection Currents, Plate Tectonics, volcanoes and earthquakes review (8th)

View Set